You are on page 1of 327
Thoroughly Revised and Updated Reasoning & Aptitude csfor GATE 2019 and ESE Pre 2019 Comprehensive Theory with oe and Solved Questions of GATE and ESE Prelims ‘Also useful for UPSC (CSAT), MBA Entrance, Wipro, $5C, Bank (PO), TCS , Railways, Infosys, various Public Sector Units and other Competitive Exams conducted by UPSC i MADE EASY a Eee MADE EASY Publications Corporate Office: 44-4/4, Kalu Sarai (Near Hauz Khas Met Staton), New Delhi-110016 Exmal:infomep@madeeasyin Contact 011-48124660, 8860376007 Visit us at: ww madeeasypublications.org Reasoning & Aptitude for GATE 2019 & ESE 2019 Prelims © Copyright, by MADE EASY Publications. Allright are reserved. No part ofthis publication may be reproduced, stored in or introduced into a roltieval system, or transmitted in any form or by any means (electronic, mechanical, photo- copying, recording or otherwise), without the prior writen permission of the above mentioned publisher of this book. 1st Edition 2008 2nd Edition: 2009 3d Ealtion :2010 4th Edition :2011 Sth Edition : 2011 ‘th Edition: 2012 7th Elion :2013 Bth Edition :2014 9th Edition: 2015 10th Edition: 2016 Mth Edition :2017 12th Edition : 2018 MADE EASY PUBLICATIONS has taken due care in colecting the data and providing the solutions, before publishing this book. lspite of his if any inaccuracy or pining error occurs then MADE EASY PUBLICATIONS. ‘owes no responsibilty. We al begratefulif youcould pointoutany such error. Your suggestions willbe appreciated. PREFACE | have immense pleasure in placing this edition of "Reasoning & Aptitude” before the aspirants of Competitive Examinations. The book has been written to meet the growing requirements of candidates appearing for GATE, ESE Prelims, UPSC-CSAT, SSC, various Public Sector Examinations, Bank (PO), MBA Entrance Exams, Railways and Campus Placements of Software Companies etc. The comprehensive volume would enable the readers toacquire complete and detailed understanding of “Reasoning & Aptitude’. It covers all dimensions of Arithmetic, Algebra, Geometry, Reasor 19 and Data Interpretation. My first-hand experience Of coaching the students has been a great source of inspiration and has helped me immensely in writing this book. Preparation for Civil Services Examination taking Mathematics as optional subject also helped me sharpen the ideas and arguments developed here, | am grateful to my parents and family members, who have been showering their blessings from the very beginning, | offer my deep sense of gratitude to my Teachers, Principals of Navodaya Vidyalayas and Professors of NIT Raipur for their blessings and guidance. | would like to acknowledge the encouragement and useful guidance Provided by my colleagues and seniors serving in IAS, IF5,1PS and IRS etc. My publisher Mrs. & Mr. B. Singh have been a constant source of support and encouragement. My special thanks to the entire MADE EASY team for bringing out the book at the earliest in the hands of readers. Suggestions and constructive comments from the readers for the improvement of the book are welcome, Nem Singh (indian Revenue Service) << Contents nis “Pages Section-A: Arithmetic 1.1 Number System 17 1.2. Percentage oo 18:31 1.3. Profit and Loss. 32-42 4.4 Simple Interest & Compound Interest 43-52 1.5 Ratio and Proportion 53-61 1.6 Average 62-71 1.7 Time & Work, : 72-87 1.8 Time Speed and Distance 88-105 (Seefion:B: Algebra & Geometry 2.1 Surds, indices & Logarithms. 107-115 2.2. Progression. 116-129 2.3 Permutations & Combinations. 130-141 2.4 Probability, 7 142-152 2.5 Set Theory. 153-159 Section-C: Reasoning & Data Interpretation 3.1 Blood Relationship. - 161-167 3.2 Coding and Decoding 168-171 3.3 Cubes and Dices 172-478. 3.4 Direction Sence Test : 179-187 3.5 Line Graph. : 188-193 3.6 Tables 194-197 3.7. Bar Diagram 198-206 3.8 Pie-Charts. a 207-216 3.9 Miscelineous Puzzles. vs 217-226 3.10 Logical Venn Diagrams. 227-232 3.11 Analytial Reasoning 233-243 _Seotion-D: Previous GATE & ES! Previous Years GATE Solved Questions (245-317, Pre 1us Year ESE Prelims Solved Questions 318-322 Section Arithmetic a Number System ive Aptitude (QA), Number System is one of the module which is of critical importance. We can consider this module as the back bone as well as basic foundation and building block for QA as wel asoriing Applications of concepts of numbers can be easily found in puzzles, reasoning bas ions, number series and ‘oning areas. This is why itis our suggestion to students to understand pts discussed in the with understanding of applications. In Quant many more module thoroughly alongs Classifications of Numbers Complon Narbe [a+ fab © eal eaeemeet cu tao pue tO OES -—__—$ —_ Fane re [el ee a Ip atelenge to oe 2a igus Fractions I fal else} ines eaner pene fis (orp? a eqs ta} i i repos : reg OME lg SS Our main focus in this module of numbers in on real number system. How ever in context, of imaginary number only following property is im lant 2 | © Reasoning & Aptitude imaginary Numbers i= Ks Fei > Ke tape Question: What is the value of expression. ii? +i $i fa) ® (o) -1 {c) 1? {d) None of these Ans. (d) Solution: (isis? 4?) Bis) If we commit a mistake of cancelling out common terms in numerator and denominator options a,b, call one correct hence my answer should be (d) but expression 1 +i 424i =14it)+G)=0 hence expression in question leading to undetermined sf] srecoretennroe Real Number System Entire real numbers group of rational and irrational numbers combined forms the set of real number, which is represented by symbol — R. All real numbers can be represented as points on a real number tine Rational Number All the numbers in piq (q # Ojform are rational numbers [p, q are integers] set of rational number is represented by — Q. Rational Numbers Follow Following forms of representations. (a) Terminating decimal forms for example 0.125 125 3 0125 = = Rational MADE ERSY (©) Nonterminating but recurring decimal forms () Forexample = 037373737 1000 = 37.73737. 990 = 37 = Q = 37/99 => rational (i) For example = 037292929 1000 = 37.292929 100000 = 3729,292928 99000 = (8729-37) oe (22087 = 9900 = © form = rational = G form = rational Fraction All rational numbers in which |q| #1 comprise the set of fractions. Proper Fractions uw Ipl lal than fractionis improper traction fraction is <~1 or more than (>) 1 alve of improper Mixed Fraction Just a modified form if improper fraction. 43 co 4 i = 3 Integers The set of all rational numbers in pig form{|q| = 1) is called as integers. It is denoted by 12 [sos -B,-2,-1,0,1,2, 3, ad it includes Negative Integers Fs (..-7, 6-5, -3, -2,-1h Positive Integers B5[1,2, Besse} Note: Status of 0 (zer0) is neutral neither positive nor negative, sy mADE EASY » Number Systems 3 Example 1 ‘There are two, 2-digit numbers ab and cd, bais the ees | aan as anather two digit number prepared by reversing the {Negative Integes ¢1) eure (Set of postive intege I) digits of ab, if ab x od = 493, ba x cd = 2059, what is value ‘g’ sum of (ab + cd) = ? Natural Numbers @ a 0) 45 ‘All counting numbers or set of positive integers is (o) 47 4 considered as set of natural numbers, I denoted ‘Ans: (d) bysat Nort] Value’! = ab x od isos N= 2.8,4 ot Itmeans ab and cd both are odd, Whole Number Hence there sum must be even, only one option is ‘Set of all nonnegative integers are considered as there which even. Hence answer is option d. whole number; it is denoted by set Example 2 W= (0,123.4 4 Note: If terms “numbers” is used without any | have multiple gift vouchers of value, qualifier than it means natural number hence forth. Rs, 101, 107, 111, 121, 131, 141, 151, 171 athe Ihave to pick exactly 10 vouchers to make payment of Rs. 1121. In how many ways | can do that? Even Numbers & Odd Numbers (a) one ee 4. Even Numbers (c) morethantwo — (d) none of these. e of All numbers divisible by 2 considered as even ‘Ans. (4) i fn Reasoning is very simple fl add 10, odd numbers FF— their sum will be always even. Hence there isnot in entire integral number line. Hence way to accomplish this. [-2, -4, -6, ...] are even integers but they are not even numbers Prime Number & Composite Numbers foper 2. Odd Number All numbers not divisible by 2 one od. Prime Numbers [1,3,5, 7, ...] are odd numbers. Number which are perfectly divisible either by 1 or L 5, -3, -1 ....] are odd integers. by itself only are called prime number. Then 25 prime number are there which are less than 100. 2 is the . only even prime number. Al prime numbers greater Properties of numbers based on even & odd than § can be expressed as (6K + 1) (K€ N) form Even + Even = Even but all the numbers in form of (6K + 1) form are not Odd + Odd = Even necessarily prime. Odd + Odd + Odd = Odd leg Caio oa Composite Numbers Odd x Even = Even Allthe numbers which can be factorized into multiple Evens eran eine prime numbers are called composite number. (Even) Even PS, Number (1) one is neither prime nor composite, (Oda) = Odd How to check whether given number is prime or (Even) = Even not? (Odd) = Odd 1. Take the square root of number These properties can be used extensively to find out 2. Consider the prime numbers, stating rom to ssitive Alternative method to get answers quickly with the help til number. Take all prime numbers upto this of options. Here are few examples. ‘square root value or nearest higher integer. 4 | © Reasoning & Aptitude MADE EASY 3. It number is divisible by any of these prime numbers. Then number is composite. Learn it by example: Suppose | want to check, is 629 prime or not? Square root of 627 is just more than 25. Then prime no. til 25 are 2, 3, 7,5, 11.13.17, 19, 23, 29. 629 is not divisible by 2, 3. 5, 7, 11, 13 but is divisible by 17 Hence itis not prime number One more example: 179 Square root of 179 ismore than 13. Hence Ineed to check divisibility of 179 against 2, 3, 5, 7, 11, 13.17 179 is not divisible by either of these hence itis a prime number, Test of Divisibility 1. Divisibility by 2 Anumber is divisible by 2 if the unit digit is zero or divisible by 2. Eg.: 22, 42, 84, 3872 ete. 2. Divisibility by 3 number is divisible by 3 ifthe sum of digit in the number is divisible by 3 Eg: 2553 Here 2+ 5 +5 +3 = 15, which is divisible by 3 hence 2583 is divisible by 3 3. Divisibility by 4 ‘A number is divisible by 4 if its last two digit is divisible by 4 Eg: 2652, here 52 is divisible by 4 so 2652 is divisible by 4 Eg.:9772, 684, 904 ete. 4. Divisibility by 5 Anumber is divisible by 5 ithe units digit in number is0or5. Eg,: 50, 505, 405 etc. 5. Divisibility by 6 A number is divisible by 6 if the number is even and sum of digits is divisible by 3. Eg.: 4536 is an even number also sum of digit 4 + 54346 = 18 is divisible by 3. Eg: 72, 8448, 3972 etc, 6. Divisibilty by 8 ‘A number is divisible by 8 if last three digit of itis divisible by 8. Eg.: 47472 here 472 is divisible by 8 hence this number 47472 is divisible by 8 7. Divisibilty by 9 A number is divisible by 9 ifthe sum ofits digit is divisible by 9 Eg.: 108936 here 140+8+9+3+6 is 27 which is divisible by 9 and hence 108936 is divisible by 9 8. Divisibility by 10 A number is divisible by 10 if its unit digit is0. Eg.: 90, 900, 740, 34920 ete. 9. Divisibitity by 11 Anumber is divisible by 11 ifthe difference of sum of digit at odd places and sum of digit at even places is either 0 or divisible by 11 Eg.: 1931, the sum of digit at odd place is 143 and sum of digit at even places is 3+1 and their difference is 4 ~ 4 = 0. s0 1331 is divisible by 11 HCF and LCM of Numbers HF, —tHighest Common Factory ofwo or more nomber is tre —&— greatest number that divides each one of them exactly, For example Bis the highest common factor af 16 and 40. HCF is also called greatest common divisior (B.C.D) LOM, (Least Common Multiple) of two or more number is the least of a lowest number which is exactly divisible by each of them, For example LCM of 8 and 12 is 24, because it is the first number which is multiple of both 8 and 12. LCM and HCF of Fractions Numerator Denominaiar: Where Denominator Fractions are written in form of denominator is not equal to zero. {H.CF. of Numerators) HGF of Fraction = (Cig of Denarinaiors) (LCM of Numerators) Mot Fraction = {LCMol Nurerators) LOM ol Fraction = This oF Dencminetrs) isu MADE EASY * Number Systems | 5 All Fractions have to be in their simplest from: 4, a" —b" is divisible by a + b only when ‘n' is even itis ee natural number Example: Find HOF & LOMot 5,5 and = Ex. : af —b! =(a?-b?\a? + b%}=(a—bYla +b) this (a? + b2), Hence at - b4 is always divisible by por. HOR.of 423) 1, (a +b) but a? b® will not be reece te Factors of Composite Number gitis Lome LOMoF (123) _6_ 5 Composite numbers are the numbers which can be “HOF of (2.37) 1) factorised into prime factors, or simply we can say that ahis composite number are those numbers which are not 2y9. © Important Algebraic Formulae prime 1. (asb) =a? + 2ab+b? For eg.: 8 is a composite number since it can be digit factorised into 2. (a-bj’ =a? -2ab+b® B=2x2x2 Similarly 9 is also a composite number, ie 3. (a-b)(a+b) =a? —b? peace : sum 4. (a+b) +(a-b)? =2(a? +b?) Composite number =P} xP}? xP}*....Ré here, P,P, i _— Py... P, ate distinct prime numbers and ay, hy. 2and garb a8 + 3aPb +cat? +? 1, are their respective powers. a aah? Factors of composite number = a (4, #1). (p+ Tl, +1) 7. (ab) =a9 —3a’b+ Sab? —b? For eg.: 18 = 2x3x3=2' x3? ~b? -3ab(a-b) Factors of 18 = (1+ 1)x(2+1)=2x3=6 Clearly it contains six factors 1, 2,3, 6, 9 and 18 isthe 8 a +b°=(a+b)(a?-ab+b?) Factors of other Composite numbers 6 = 2" x3! actl Factors = (1+ 1)x(1+1}=4=1,2,3and6 Sang #78 =(a-by(a? +a +b") Te 2x2 2x9x9— 2x3! 10. 2° b? +08 -3abe =la+bec) Factors = (3 + 1) (24 1)= 12 ‘isior "af +b? +0? —ab—be-ca Ex.1_ Find the factors of composite number 360 " at -b* =(a) (02) (a? +0? )(a?-b’) Sol, 860 = 2x 2x 2x3x3K5 x3e x5? ala? ab? sr Factors = (3+ 1)(2+1) (1 +1) =24 ve by — Condition of Divisibility for Algebric Function pooo 1. a" + b"is exactly divisible by a+b only when nis is the odd Ex: 346° = (a+ b)(at abs) is cvsible by Counting Number of Trailing Zeros + P+ DS is sit + 2 = pee & + bis divisible by arb ‘Sometimes we come actoss problems in which we have 2 + bts never divisible by a-b (whether n is odd to count number of zeros at the end of factorial of any where oreven) Ex. a? + b? = (a +b) (a?—ab + b%) isnot divisible by (ab) a + b7 is also not divisible by (ab) 3. a" bis always divisible by (a~b) (whether nis odd or even) Ex.: a - b° is exactly divisible by (a—b) also al? bis also exactly divisible by (a—b} number. For example Number of zero at the end of 10! 10! = 10x 9x8x7x6x5x4x3x2x1 Here basically we have tocount numberof fives, because ‘mutiplication of five by any even number will result in 0 al the end of final product. In 10! we have 2 fives thus total number of zeros are 2. 6 | Reasoning & Aptitude MADE EASY mE Short Cut: Counting number of zeros at the end of n! na Value will be 2+ 2 lve will be ey oF The integral value of this sum will be the total number of zeros. Ex.1_ Number of zeros atthe end of 100! 400 , 100 , 100 ee 58 8 integral value wil be 20 +4 = 24 zeros, Sol Ex2__ Number of zeros atthe end of 126! 126 | 126 | 126 e integral value willbe 25 +5 + 1=21 20008. Sol. gooo Cyclicity Cyelicity of a number isused mainly for the calculation of unit digits. vycticity of ae In 1°, unit digit will aways be 1 2. Cyclicity of 2. 222 Bed 2-8 28=16 2 = 32 2 = 64 27 = 128 28 = 286 Aiter every four intervals itrepeats so cycle of 2is 2,4, 8.6 Ext Unit cigit of 22 Sol Here 2, 4,8, 6wilrepeat after every four interval till 320 next digit will be 2, 4, [8]. So unit digit of 28 will be 8 Ex2 Find unit digit of 1212 x 222 Sol: Unit digitof 12" willbe 6 and 22%? willbe 4. $o unit digit of 12°22 will be 6x4=2[j:4 Ans. 3. Cyolicity of 3. 1. Blag a9 3527 3tset 38 = 243, = 729 37 = 2187 3° = 6561 After every fourintervals 3.9, 7 and 1 are repeated, So cycle of 8i83,9, 7, 4 Ex. 7 318 Ext Findunit digit of 138 Si Sol: Cycle of is 3, 9,7, t which repeats ater every four intervals tli 193°, So next unit digit will be. 4. Cyolicity of 4. ae #a18 464 4h = 256 Cycleis 4,6, ie. Unit digit of 4” depends an value of n I nis odd unit digit is 4 and ifn is even digit is 6. Ex 028 Sol Sol: Since 1024 is even number unit digit will be 6. Ex.2 Find unit digit of 133% x4 Sol: Unit digit of 183° is 7 and unit digit of 4s 4 so Unit digit of 1338° x 4" willbe 7 x 4 = 28 ie. 8 a 5. Cyolicity of 5. 5125 52 =25 5 = 125 54= 625 Unit digit wil always be 5. Ex. 6. Cyclicity of 6. 6'=6 6=36 6216 6t= 1208 Unit digit wil always be 6. Sol Ex.1 Find unit digit of 4° x 6° Sol: Unit digit of 4% is 4 and unit digit of 6° is 6 80 unit digit of 4° x 6° will be 4 x6 = 24 i.e. 4, sy ed.So 7 “every git will gitis6 ibe. is 450 sie. 8 is 6s0 mabe ERSY * Number Systems | 7 7. Cyclicity of 7. Remember Ne? Cyclicty table 1H 2 :2,4,6,8 3:3,9,7,1 4:46 5:5 6:6 = 8764801 a a 7.9.31 eee ee 8:84.26 Ex1 Find unit digit of 17"? «2727 9:91 Sol: Unit digit of 17” is 7 and unit digit of 27°” is 3 0:0 So unit digit of 17°" x 272? will be 7 x3 = 21 ied ooo 8, Cyclicity of 8. Remainder Theorem So cycle of Bis 8, 4, 2, 6. Ex. 1 Find unit digit of 18° x 28°8 x 28828 Sol: Unit digit of 18'°is 4, unit digit of 2828s 6, unit digit of 2882 is 6. So unit digit of 181° x 28%x 268% willbe 4x6 x6 = 144 i.e. 4 9. Cyclicity of 9. =9 9 = 729 9 = 6561 Cycle of 9 is 9, 1 in 9" unit digit willbe 9if mis odd and unit digit wll be 1 if nis even. Ex.1 Find unit digit of AN 4 1212 4 135% 4 1414 4 1518 Sol: Unit digit of 1111 is 1 Unit digit of 12°2is 6 Unit digit of 13°%is 3 Unit digit of 1414s 6 Unit digit of 15°%is 5 So unit digit of given sum will be 1464346+5=2ie. 1. axbxe n Remainder of expression [ie.axbxewhen divided by n] is equal to the remainder of expression ay XbuXCy fg a, xb,, x6, when divided by n}, where n a, is remainder when a is divided by n, by, Is remainder when b is divided by n, and ¢, is remainder when c is divided by n. Ex.1 Find the remainder of 15 x 17 x 19 when divided by7. 15x 17519 Sol.: Remainder of expression ~~" will be 1x35 equal to ie On dividing 16 by 7 we get 1 as remainder On dividing 17 by 7 we get 3 as remainder On dividing 19 by 7 we get 5 as remainder and combined remainder will be equal to remainder of Sie 7 ooo 8 | © Reasoning & Aptitude MADE EASY Polynomial Theorem nx LOM of (4, 5, 6)- 1, = 60n= 1 Win = 41, 60-1 = 89's smallest such natural This is very powerful theorem to find the remainder. number, ar ‘According to polynomial theorem. Note: n such numbers are possible Here we have taken (4 aff = a+ °C, x1 4 90, x2 a? ‘has 1. Other numbers are 119, 179, 239, ete, Where 5, 4°Cya909,. °C, yale a? value of nis 2, 3, & 4 respectively. hone mt a oe Ex1 On dividing anumber by 5,6 and 7 wegel3,4 Se Tyas 4MCy xT al 4 Cy x? a? + and § as remainder. Find the number, (xtayt [Cy 29+... Cay stale Sol x 5 6 7 remainder of expression (2) will be equal to Remainder 3 4 5 a which is equal to (5-2), (6-2), (7-2) remainder of “— because rest ofthe term contains that number will be: are completely divisible by x. nx LOM of (5.6, 7)~2= 210 ~2 = 208 oe Note: Here we have taken value of n as 1 Sq a ele aE Fx2 Ondividing a number by 4,5 and6weget2,3 °F wo . and 4 as remainder find highest possible three |‘ eo oe oa digit such number. é 8 Sol. ‘According to polynomial theorem remainder will 4 5 8 p Remainder 2 38 4 be equal to remainder of the expression — which is equal to (4-2), (5-2),(6 - 2), that : number will be: ae x LCM of [4, 5, 6-2 =n x 60-2 When n = 1 we get 58. Highest possible three Ex.2 Findremainder of © digit such number will be 958. ayy Ex3 Onuividiny anumbe by 5, 6and7wegetS.4 sol; 5”. (4) = and 5 as remainder. Find highest possible three u digit such number. ee a Sol. 2a 5 6 7 = Remainder is 2 feoteeeet ne which is equal to(5-2), (6-2)(7 ~ 2) that number willbe More on Remainders n+ LCM (6, 6, 7)}-2=nx 210-2 Highest possible three digit number willbe 638. Case-! Case On dividing a number by a, b & cif we get a-k, b-k and c-k as remainder respectively then that number will be nx LCM of [a, b, c}-k. For ex (I); On dividing a number by 4,5 & 6 we get 3, 4, & 5 as remainder. Find the number. Sol.: 4, 5, 6 Remainder 3, 4, 5, which is equal to (4-1), (5-1), (6-1), so that number will be: On dividing a number a, b and c if we get k as remainder always, then that number will be (n ~1) LCM of (a,b, c}+ k Ext Ondividinganumberby 6, 6and 7 fweget2as remainder always, find that number Sol: That number will be (n~1) x LOM of 5, 6, 71+ 2 =» 2 is such smallest number next number will be = 210 + 2 = 212 yalual | | a number after adding k is exactly divisible by a, b plan | NXLCM(a,b, c)~k Where" Ex.4 Find a number which atter adding 7 is divisible by 10,11 and 12, ye3.4 Sol: That number wil be nx LOMof[10, 11, 12]-7 n= 1 then 660-7 = 653 Ans onoo Squares of Numbers 1. Squares of numbers are frequently used for calculations yet2,3 0” various types of problems. It is advisable to a thvee | femember square of a least fist thirty numbers bet 1 = 121 Bag 12 9 12 16 1 ), that 3 is x 162 = 256 49 1 e thvee eA 18 81 19 a ea a three POM following table we come to know that square of a 2) that 20 838. atk as vill be yel2as MADE EASY Case-IIl and c then that number will be umber always ends with 0, 1, 4,5, 6 & Sas unit digit Square of a number can never have 2, 3, 7 & Bin its Unit place, On observing squares of numbers between 21 to.23 we Get following pattern 2 eda ata] 23 S29 24° = 5|76| 25° = 6/25] Last two digits are common Observation Square of two digit number having 6 in unit places can be calculated very easily "Shere n may 1 109. (nS)? = [ns (n+ 1)]25 * Number Systems 9 Ext 65%= Sol [6x (6+ 1)]25 = 4225 Ex2 85=7 Sol: [8x (B+ 1)]25 =» 7225 Ex3 96%=? Sol: (9x (9+ 1)]25 = 9025 Base System The Number system is used to represent any number Using aset of symbols (digits eters). The base defines the number of symbols in particular base system, We generally work in Decimal system as there are 10 digits (0, 1,2.....9), Some others systems are; Binary base system: 2 symbols: 0, 1 Octal base system: 8 symbols: 0,1,2,3,4.,6,7 Hexadecimal system: 16 symbols: 01,2,3,4.5,6,7,89, A= 10,B= 11,0 = 12, D=13,6=14,F = 15 Converting any number from any Base system to Decimat number system: abe efgy = axB°+b xB +oxBl+dx Bo +exB' +ix824gxB9 Example: 1234.56, 1x89 428743 x Bl 4 4 xB? 49x81 46x82 124 128+ 244 4 + 0.625 + 0.093750 = 068.718750 Converting any number from Decimal to other Base system: Divide the number by base and gel the first remainder 1, and Quotient q, Now divided q, by base and get remainder r, and Quotient g, Repeat the folowing process till we get the quotient y= 9 Now the decimal number in base b is fafa Example 1 1 (149 90=()y 7149 [Remainder 2 7 a Osea (149) = (802), 10 | * Reasoning & Aptitude MADE EASY mi 2. Add(432), + (355), Ans: (d) 9 M1 ecarry LOM of numerators (a0, LC of fraction = nates (355), Here, 5/2, 8/9, 11/14, so a0 as 2+5=(T)yq= (10), OM eS ey 34541 = ()jg= (12), : 14443 = (8g = (1) Find the number of divisors of 1420. gaua fa 14 (0) 15 © 13 (a) 12 Ans: (d) 1420 = 142 x 10= 71x 2x 2x5 = 2x6! x71! | Solved Examples No. of divisor = (241) (141) (141) = 12 10 Amilkman has three different quaities of mi, 403, gallons of 1" quality, 465 gallons of 2 quality and 1. The sum of the digits of a two-digit number is 10, 496 gallons of 3 quality, Find the least possible while when the digits are reversed, the number number of bottles of equal size in which different decreases by 54. Find the the changed number. milk of different qualities can be filled without (a) (0) 19 mixing? © 7 (a) 48 (a) 4 (©) 46 Ans: (a) (0) 2% oa Going through options we get 82-28 = 54 Ans: (d) 4 2. The sum of two numbersis 15 and their geometric aca ‘nai gatons . mean is 20% lower than thei arithmetic mean. Find Sent : ania ” (b) 12.3 ae ee fa . least number of bottles will bein size of HCF (403, ( 132 ee 485 and 496) peal 403 = 13x31 Gong trough options only 12and 3 satisfices the ee 496 = 16x31 awel2#3 75 HCF = 31, Sowe required 13+15+16= 44 bottles 4p : What is the greatest number of 4 digits that wher GM =V72.x3 = 6V3 whichis 20% less than 7.5. divided by any of thenumbers 6,9, 12, 17 leaves¢ remainder of 1? 3. If A361 is divisible by 11, find the value of the (a) 9907 (b) 9793 smallest natural number A? (©) 9605 (a) 9487 (@) 8 0) 6 Ans: (b) ong (do LCM of 6, 9, 12, 17 = 612 Ans. (c) greatest number of 4 digit divisible by 612 is 9792 ‘A381 is divisible by 11 ifand only f(A + 8)-(3 + 1) to get remainder 1 number shouldbe 9792+1 4.4 is divisible by 11 ‘ So, AaT Satisfies the condition Which of the following is not a perfect square? (2) 100858 (b) 3, 25, 137 4. Find the LCM of 5/2, 8/9, 11/14 (©) 945723 (4) All of these “i (a) 280 (b) 60 Ans: (d) ©) 20 (A) None ofthese ‘Square of number never ends up with 2, 3, 7,8 ERSY mADE EASY * Number Systems | 11 9. The LCM of (16 ~ x°) and (x? +.x— 6) is Ans: (c) @) (= Siee+ 3)(4 — 7) GOD of (x43)(x-2}(x+ 1 and 5 (b) 44-243) : (@ (4-2-3) (x+ 9° (x+3)(x+4) will be (243) (x + 1 (@) None of these : Ans: (d) 14. Find the LCM of (x + 3) (6x? + 5x ~ 4) and : (2:2 + 7x +3) (043) 16-x° =(4~x)(44x) (@) (2x +1) (+3) Bx + 4) (0) (4x2-1) (x + 3)? (3x + 4) ( +x-6)=(x+3)(x-2) (©) (ax2~ 1) @ +3) (3x + 4) {d) (2x - 1) (x + 3) (3x + 4) Lom will (16-2°)(x? +x~8) Ans: (b) xm 2 45-4) = ~Naxe 10. GCD of x2 4 and x2 + r-6is (3#3)(61? +514) =(043)(20—Y(3x+4) (@) x42 (0) x2 . milk. 403 @P-2 (@ x42 (2x7 + 7x +3)(x43) = (e+ Y(v+3)(r43) salty and ‘Ans: (b) , possible 7 LOM= (2e-+ 1)(2r~ (x43) (Bx+ 4) different 2° <4 =(x~2)(x+2) 4 without = [4x2 = 1)(x+3)° (3x44) (2 12-6) (e+9)(e-2) (4 Hesaftared) 15. The product of three consecutive natural numbers, GOD =(x-2) the first of which is an even number, is always 11, Decompose the number 20 into two terms such that ree by oe their product is the greatest. Ba {@) Alot hese (a) x,=x)= 10 () x,= 5,42 15 ec (©) x)= 8.2,= 12 (a) Noneof these Three consecutive number will be n(n + 1) yor (403, Ans: (a) {n-+.2) if nis even number then {n + 2) will alsobe Hey constant tency willbe maxeTuan ane an even number and one of them will be divisible rey by 3. Hence number is always divisible by 12, here, = 20 x 46. Find the pairs of natural numbers whose least common mulipleis 78 and the greatest common “A bottles 42, For a number to be divisible by 88, it should be divisor is 13. that wher (a) Divisible by 22 and 8 (a) 58 and 130r 16 and 29 Tleavest (0) Divisible by 11 and & (b) 38 and 23 0r36 and 49 (c) Divisible by 11 and thrice by 2 (6) 18.and 73 0r56 and 93 (d) Both (b) and (c) (d) 7B.and 130126 and 39 Ans: (b) Ans: (d) Anumber to be divisible by 88 it should be divisible LCM = 78 and GCD = 13 by Sand 11 because 8 and 11 are.co prime numbers. Clearly 13, 78 and 26, 29 are the two numbers 1 2 whe tipi cee ose multiplication gives 68 17. Fill in the blank indicated by a star in the number 2 18. Find the GCD of the polynomials (x + 3)? 4° 56 so as to make it divisible by 33 quare? (= 2)(r4t)Pand (+ 186 +3) («4 4), f@) 3 (o) 4 (a) (x +35 + 1)? (x 2)(x 4 4) (5 (d) None of these (b) (+3) (e~2) (e+ 1) (r+ 4) Ans. (a) (©) (43) (e412 4°56 is divisible by 33 if and only if itis divisible by soe (d) None of these Sand 11. 12 | _* Reasoning & Aptitude MADE EASY mA 4° 56 will be divisible by 3 if * will be equalto 0, 3, (a) Rs 1200 (o) Rs. 1400 6.9 (©) Fs, 1700 (A) None of these 4°56 is divisible by 11 (4 + 5)~("+6) willbe Ans. (a) divisible by 11 s0* should be 3 One alternate method 18. Find the least number which being divided by 9, Let Ihavexrupees 12, 16 and 30 leavesin each case a remainder 3? 1 ox * @es ‘b) 728 Alter spening 75 of ton house rent have 2 (©) 728 (@) None of these ‘Ans. (b) 9x 1 9 LCM of 9, 12, 16 and30is 720 so required number Now out of fiabse7 a6 28.N = n(n + 1) (n+ 2) (n + 3) (n + 4); where nisa = =n natural number. Which ofthe following statements is/are true? Alternative Method 1. Unit digit of Nis 0 2. Nis perfectly divisible by 24. a60 +2. :) 3. Nis perfect square. 9+ 4/5 4, Nis odd. = ayei =27 (@) 3only (0) 8and 4 only 380 = set (©) tonly (d) 1 and 2 only and 34 944V5 24, How many factors of N= 12" x 147 x 15° are multiple of K = 1210 x 1490 x 150 (a) 2x4x8 (0) Bx7x4x5 (0) 3x5x6 (0 9x8x6x5 3 Thus, [3/80+—". = Vat “ovale © YN = 3x9 = 3V3 ERsy, mAape ERSY ® NumberSystems | 15 2. (a) 6. (b) Use plugging in by 2 | ' -om as (x+3)8-8 = 117 P= yleven x4325 as (v+3)2117 = ¥aeven ee = S82 125-82 117 xis int so only ay must be even. So, sum of both numbers = 7 Aiternative Method 3 (e) 1 (498-23 = 117 a 3+) +3) @) + 3)-282 117 ae 427 4 20n4 02 —8= 17 = Ox? 4 274-90 = 0 50 terms... sonnI TA = x24+3r-10=0 0 = (e+5){x~2) =0 nit digit (11... 50 times}= 0 a and carry =5 * alization —_ tens digit (1 + 1 +.... 49 times)+ camy 5 = 4 = ee 5 d ia ‘i 7. () Ne Putin prime factorization theorem, . oy" = (65) @ 28x = ey = ae" Total factors = 6x6 x4 24 as am =a" fb) 27x (2x3 = 2x3? and 2-10.41 Total factors = 14x 424 = men © so fy =-3r (@) 68x85 = RX 71S TT 4 3x2x2x2=24 ety 8. (0) 6. For remainder we have to caloulate the unit digit gt” aol =o 14453 Thus, 3%*® gives unit Digit =» 3x3x3=7 3” so, Rem Thus remainder is 2. As or checking divisibilly by Sis checked by dividing last digit of number. N= 129 18%x 14 (22 x 3) x 13? x (7 x2) 2F 397! x 192 Number of factors, (741) B41) (14 1241) = 192 a 34 =10 16 |» Reasoning & Aptitude MADE EASY m 9 (b) 13. (c) 18 1421 1428% 142 Ret Fon] Metz. raaes 2 [5x7x9] [5x9] 3 Rem = Rem’ Se [x9]. (99) Remi 5 = Remi) «3 « 14, (b) ron $5] -hen(2) «3 nes 298 New 7 = SAIS _ 5x, MES 10. (a) 7 N= (114214314 41+...1000!) =5kK +245 Now we have to check only "7 1! 421 431 + 4! as after that will factorial has unit eae digit as 0 15. (d) 51 = 120 Number are 105, 111...195 6! = 720and soon . Thus unit digitof (1! +2! +3! + 4! + 0) forall other = Total 29-19 04 6 > (142464440) Unit digits ofall factorial as number is odd and not mutple of 7, 20 = = Thus in total 16 number there are 3 numbers 33x3x3x9=1 unit digit. Hence when 21x5 = 125 (8)"°-+ 10, Remainder will be 1 2x7 = 147 21x9 = 189 " o ch lysunioh enous which are multiple of 7 socioeeueee es Thus 16-3 = Wnumbers between any two number at s0 number may be (5r,, Sr, 813.53, 16. (d) and HFC (yi, x))= 1 Suppose division ‘a! and remainder ig"? Thus, number are 3 MOM = a, d+r (8% 1,8 2,6%3,5x 5...) and 32508 = qd 41 because (x, x) = 18365 = (q,-9,)¢ Then only prime number will work. = d should be factor 1535 80 §x 19 = 9Sis biggest number. So, only 307 is factor of 1535 in given option, 80 5x1,5x20x96x5,5x7SXEX1RS 47 (gy 22 x 17,5 x 19, total 9 number. £8. 459, 467) 12. (d) This san Arithmetic Progression Plugging in 467 = 3+ (n~1)8 @) ystQx= Total n= 59 =10-2=8 x Now, (3, 467), (11, 489), (19, 451). xt » . Total 2 = 29pair ee All will make sur 470, ta deteaee x +2etsana (©) w+y)= 1042512 X (d) None of above is true. ‘Sowe can take only 1 elements from each pair. ‘Thus (otal elements in T can be fone of 3, 467), one of (11, 459)...} Total 29. Sy mADE EASY © Number Systems | 17 tion. 2h pair, 18. 19. 20. 24 22, (c) (@) 100=5 x2 possible (0) 12=6 x 2 possible x 5 not possible asonly one § Number (@) 90= 15 x 2 possible () Suppose gph oh aN A = Natural Number 4 wm Masta N 24 Tusto be Mas an natural number $7 should be Natural number thus N should be factor of 24. so 24 = 23x3" Factors = 4x2=8 8 values can be taken by N (a) Unit igit of 299 < 072% x gol? Unit digit of (95% x 7% <6") = (HK 5 74403 yee) = (@x3x6) units digit (a) Number of zeros can be calculated by getting power of 10 in number or minimum among powers of 205 possible in. Thus 2x 38K ou 97 soonly 15's and 12 Thus only one zero as both 2 and five will occur only once, (c) 123....910. One digit numbers (from 1 to 9) = 9 Now 2 digit number til 50 are 50 ~ 9 = 41 thus {otal digits used to make pair. WO AL 1229. So total 20 x 2 = 40 digits would be required to write numbers from 10 0.29, Total digits consumed so far tll we write 29 will be 9440 =49. 50" digit which we will write is 3 of 30. Hence 50 digit will be 3, 49 digits are used. So far til 29. Now, 60" digit would be frst digit o 30 thus 3. (d) N= nine 1) (n+ 2)(n +3) (n+ 4) is product of 5 consecutive number. Thus divisible by 5! = 120 So, (i) unit digits 0 TRUE, (ji) Pertectly divisible by 24 TRUE. (ii) Nis perfect square not TRUE. (iv) Nis odd X not TRUE, Thus (4) (i) and (i only (a) N= 1212 14" x 1515 22 x3)! x (7 x 2]! x (5 x 3)" = 22 SIT 214 x5 x3" 288 327 518 x74 = (12)! x (14)79x (15) 1220 9. 310 710 DID y 310 510 = 220.3% 51x 710 Thus factors of N which are multiple of k are 290 x G20 x 510 «710 (28 x3! x 5° x74) will be (B + 1) x (7+ 1)x (5+ 1) x(4 + 1) 9xBx6x5 (b) 13% hy 402, (7+ 4)=(11),0= (2) unit digit is 2 thus 9 is possible base ‘Thus we have to add using base 9 50 342, 562, i014, (b) For remainder by § we have to calculate the unit digit only For unit digit of 7”” get (7)"**F from = (ye = (+1) Thus (3) Remainder a7 ark = unit digitis 7x 7x 7=3 3 So Rem} =|=3 s0(b) CHAPTER The term ‘percent’indicates the value out of hundred This concept of percentage is developed to make the comparison of ratio easier by taking the denominator value as 100) The concept of percentage is very usetulin reasoning & aptitude and specially for data interpretation section where every logic that has to be used, is based on percentage, Calculation of Percentage As we know that percent value is the value out of hundred. The percent value is calculated as Value Mey Total Value os The basic thing that has to be kept in mind is that the value in the base must be taken care of. Percentage & Fraction The value percentage can be represented in three different form each form is important for making culation simile, Here are some percentage values given to understand different form TetomT ti Multiplication factor Multiplication factors the value by which a particular quantity has to be multiplied to show the final changed value. Itis nothing but fractional or decimal form of percentage: mi Percentage St Iwhas to be understood in this manner, that a 6 3 jantity is 200. Its 60% value will be ==, =, 0 quantity is s 60% value will be <5, 5,06 : times of 200. Here all these values are the multiplication factor for the quantity 200. Multiplication factors are very important when we talk about the percenrage increment or decrement of a quantity. Let us understand with some go examples. Example 1 A quantity 200 is increased by 75% then wi be the new quantity? Solution, One thing we can do is to find out 75% of 200/.. 5 =x 200 = 1 3a = hen the new quantity will be 200 + 150 = 350 we can move in ather way as the quantity inilily was 100% i.e. 1 as a multiplying factor. Now it increased by 75% ie. 3/4 oF 0.75 as a multiplying factor. So the new quantity becomes (1+3}x200 = 4 & (140.75) x 200 = 350 3 These (: +3) and (1 + 0.75) are the multiplying factors, Let us see one more example to understand it clearly. Example 2. A quantity 300 when increased by some percenrage becomes 360. Find cut by what percentage it has increased? Cc Solution. Now we know that 300 x Multiplying Factor (MF) = 360 we the} hen we, rement some ‘hat will 200 1.0 0 = 350 rinitially low itis tiplying tiplying erstand senrage jeithas * Percentage | 19 mao EASY : - 369 sa & gee MF = 309 5 ‘This shows thatthe muttiolying factor 1 is increased © by 0:2 £ hich s equal to 20%, sonet increment percentage is 20%, Successive Percentage Change Successive percentage change is one percentage ~ change over and above another percentage change. Let us take one example to understand it Example 1. ‘A quantity 300 is increased by 20%, Then it was decreased by 10%. Find out the new quantity is how much percentage more/less than 300? Sol 300 when increased by 20% becomes 300(1,2) or sxa(1+2) Now this new quantity is decreased by' 10%. So the final quantity will be [300(1.2)K1-0.1) [m(5)(-a]] = 00(1.08) 4 300f 1+ or of + 4) 4 Sowe can understand the net result is 0.08 0" = increment in MF. Whichis equal to net 8% increment inthe quantity 300, So it will be very clearly understood from the example that in successive percentage changes, the multiplying factors are multiplied directly. So the point that has to be understood is “in case of successive percentage changes the net multiplying factor is the product of all the corresponding multiplying factors Let us take another examples as: Comparison Leading to the Base Change Sometimes whenever the percentage comparison. Occurs between two different values. The base change occurs. Multiplying factors are again very Useful here. Let us understand with some examples. Example 1 The salary of Ram was increased by 40%. But Ramis performance kept on declining. Because of which his employer decreased his salary to the salary before the increment. Find out what percentage deduction was provided by his employer? Solution. Let us assume in starting salary of Ram was x and after the increment it became y. Now his salary was again decreased from y to x and we need to find out this percentage change, we can say itas =(+8)- Y= ("F99, 2 = y=(142)s v=[¥§) = yn ey 5 So we can say the % decrement in the salary of 2 Ram is the percenrage equivalent of 5 which is 2 100% io, 28.56% Example 2 Rama uses rice as his daily meal. But because of inflation the price on rice were increased by 10%. Find out by what percentage he has to decrease his consumption to keep the expenditure same as, previous? Solution. Letus assume initial price was P and the expenditure was’ then, Price x consumption = Expenditure = initial consumption = Now price became ub call 100)” * 70) ae eg ANP, Then final consumptio 20 | * Reasoning & Aptitude MADE EASY ° Example 6. Sofinal consumption = + inital consumption) ‘A shop provides flat 50% discount on one shir. While another shop provides the successive ire corte eine bee mt cect ement a discount of 30% and 30%. Ifthe difference of the hoy bills is Rs. 43, Find out the net cost of shit consumption will be (-2) ie. 75 part Solution. Let the price of shit be + 1 The percentage equivalentwie “x 100 =9,09% eee eee "i = (1-05) = 05 Alternately: The net percent change For the second shop the bill is a 0.7 x OFx = 0.498 = 10X16, - 9.00% The difference in the bils 2 100410 canes . Example 3. ee ‘The price of a toy was increased by 20% & thenit Example 6, was sold at 20% discount. Find out the net change Radius of a sphereis increased by 10%. Find out in the price of the toy. the net percentage increment in the volume. Solution. Solution, The net multiplying factor Net percentage increment in radius = 10% New radius = 1.1 = (1420), 22 a ; =U toa} t00, So the new volume = (1.11) SECS =(1.1}5(original volume) =U 008) = 1.331 (original valume) 7 ‘Sonel percentage change TEC pErceNtnCreMTeM = 33.1% ; = 0.04 x 100 = 4% decrease Example 7. aoe In an office 40% of the employee are males while 70% of the employees are married if 80% of male The price of a ball pen & a gel pen is same. Now are married find nut he net percent of unmarried the price of ball pen is decreased by 30%. While of female in the office Gel pen is increased by 20%. Find out the net percentage change in the net total price of both Solution. pens Letus assume total employees as 100 then Solution Mariel al] Let the price of individual pens be x Weis | ora Total price = 2° Female | 70-22-38 New price of ball pen = (1-0.3)r = O.7x Totat_| 70% of 100=70 Now price of gel pen = 1.2r New total is = 1.8x = 0.95(2x) A 22 Sochange = 5% decrement Sonnet percent 775100 = 22% nooo sswhile of male martied * Percentage | 21 C7 Solved Examples Which of the folowing is the largest number? (@) 20% 0f200 —(b) 7% of 500 (© 1300%0f3 (a) 600% of 7 Ans. (d) 20% of 200 = 40 7% of 600 = 36 41900% of 3= 39 600% of 7 = 42 Mr, Rajesh is worried about the balance of his monthly budget. The price of petrol has increased by 40%. By what percent should he reduce the consumption of petrol so that he is able to balance his budget? (@) 3333 (b) 28.56 (0) (d) None of these Ans. (b) ‘We know that % reduction required is 4400 herex=40 jo0+x 40 S100 = 28.56% 440 * . Inan election between 2 candidates, Ravikant gets 65% of the total valid votes. If the total votes were 6000, what is the number of valid votes that the other candidate Shailendra gets if 25% of the total votes were declared invalid? (a) 1625, (b) 1575 {c) 1675 (d) 1525 ‘Ans. (b) Total votes 6000 Invalid votes = 25% of 6000 = 1500 Total valid votes = 4500 Ravikant gets 65% So other candidate gets 35% 35% of 4500 =1575 In arnedical cerificate, by mistake a candidate gave his height as 25% more than normal. inthe interview panel, he clarified. that his height was 5 feet 5 inches. Findthe percentage correction made by the Candidate from his stated height to his actual height. (@ 2 (0) 2856 (©) 25 (d) None of these Ans. (a) Height increased =25% here x = 25 to reduce to normal % correction requiredis 25 =x 100 100+ 25 20% .Anumber is mistakenly divided by § instead of being multiplied by 5. Find the percentage change in the result due to this mistake, (a) 96% (b) 95% (©) 2400% (d) None of these Ans. (a) Let number is 100 Itis divided by 5 we get 20 Now actual result should be 5 x 100 = 500 So % change is result 500-20 £0020... 109 = 06% 500 te In a mixture of 80 litres of milk and water, 25% of the mixture is milk, How much water should be addedto the mixture so that milk becomes 20% of the mixture? (a) 20litres (b) (lives {c) 28 litres (@) None of these Ans. (a) Total mixture 20 lire Mik is 25% Le. 25x80 20 litre 700 tomake it 20% amount of water required to add BO+x DX 100 = 80% 6000 + 100x = 6400 + 80 20x = 400, x = 20 A landowner increased the length and the breadth ofa rectangular plot by 10% and 20% respectively. Find the percentage change in the cost ofthe plot assuring land prices are uniform throughout his plot (2) 33% (b) 35% (c) 22.22% (a) None of these Ans. (d) herex =10 ory = 20 10 suchangeinarea-(x+y+ 2) 10x20 st AS 392 0+ 20+ =a 22 | ® Reasoning & Aptitude mabe ensu 8. The length, breadth and height ofa room inthe shape of a cuboid are increased by 10%, 20% and 50% respectively. Find the percentage change in the volume of the cuboid. (@) 77% (} 75% (©) 88% (a) 98% Ans. (d) Let, b, hbe length, breadth and height ofthe cuboid Volume v= 1b. Now, 1, b, h are increased by 10%, 20% 50% respectively fre 22 ecole] onl 4 2 vee Zg| oft] 5 = 1.98 Lb. % change = 98% the population drops by 5%. Find the population at the end ofthe third year iin the third the population increases by 20%. (a) 12,340 {b) 12,540 (ce) 127,540 (d) 12,440 Ans. (b) P =10000, x= +10%, y =—5%, z = 20% Population at the end of third year (Ee SS) 100)" 700 )\'* 700 10), 5 Y,, 20 . 2M SY 22 ooo inal imo) **ica) = 12540, 1 12. In an examination, Mohit obtained 20% more marks. 9. The price of sugar is reduced by 25% but inspite of than Sushant but 10% less than Rajesh. If the marks. the decrease, Aayush ends up increasing his obtained by Sushant is 1080, find the percentage expenditure on sugar by 20%. Whatis the percentage marks obtained by Rajesh ifthe full marks is 2000. change in his monthly consumption of sugar? (a) 86.66% (b) 729% fa) +60% (0) -10% (©) 78.33% (d) None of these (c) +33.33% (d) 50% Ans. (b) Ans. (a) ‘Marks obtained by Shushant is 1080 Let price of sugar be.x& expenditure E 120 3 Mohit’s marks = 700" 1080 = 1296 Now it is reduced by 25%. So it is ae now ra expenditure of sugars also increased by 20% ie, __~Rajesh Marks, R=> 7755 xR = 1296 R= 1440 128 0° Se aso Ai So quantity of sugar that can be purchased 9% of Rajesh Marks= 95 x 100= 72% 8 * 5 13. The population of a village is 5500. I the number of 4x males increases by 11% and the number of females Hence consumption increased by 60%. increases by 20%, then the population becomes 6390. Find the population of females in the town, 10. 30% of a number when subtracted from 91, gives. eae (@) 2500 (©) 3000 the number itself. Find number (©) 2000 {d) 3500 (a) © tb) Ans.(a) (0) 70 (Oe ee xis population of male: a) (6500 -»)s female population 30 B= 90% ofmumbers rt, (6500~2)120 _ go 20 100 700 Now 91-5 On calculating we get x = 3000 oi 130, — ‘So female population = 2500. 100 44, Vicky's salary is 75% more than Ashu’s. Vicky gota 11, The population ofthe vilage of Rampur is 10,000 at this moment. It increases by 10% in the first year. However, in the second year, due to immigration, raise of 40% on his salary while Ashu got a raise of 25% on is salary. By what percentis Vicky's salary more than Ashu’s now? asu) MADE ASS ionat (@) 96% (b) 51.1% lation = (0) 90% (a) 52.1% Ans. (a) © Lets Ashu’s salary = Rs. 100 Vioky's salary = Rs. 175 Vicky's salary increased by 40% i.e. M0475 =, 245 700 Ashuis salary increased by 25% ie 125 Vicky's Salary is 120 more that Ashu's in % term. t oo 28100 = 96% Be 128 © 48. Duringwinters, an athlete can run x’ metres on one rails |», bottle of Glucose. But in the summer, he can only narts | tun0Srmetres on one ble of Gucose, How many Dos, boliles of Glucose are required to run 400 metres during summer? 2 @ 00K (b) 890! 1% @) 454k Ans. (a) During summer to run x one bottle of glucose is required 1 thentorun 1 km — bottles 5x erat 10; !Manetamination, 80% students passed in Physics, “ che 70% in Chemistry while 15% failed in both the es subjects. If 325 students passed in both the mn Subjects. Find the total number of students who appeared in the examination. (a) S00 (b) 400 (©) 300 (@) 600 Ans. (a) 0% P on 70% i 15% student failed in both subjects onl So total passed = 85% al 80+ 70= 854x, x = 65% sary 65% of total = 325 i total = 600. * Percentage | 23 17. 40% of 20% + 30% of 25% + 50% of 28% is equivalent to (@) 295% (b) 28.5% (©) 305% (d) None of these Ans. (a) 40,20 _ 8 40% of 20% = 35% 455" Fog 5, = 20 25 30% of 25% = 5x55 50, 28 94 of 28% = 20» 28. and, 50% of 28%4= EO a4 n1a% 700 +. (40% of 20% + 30% of 25% + 50% of 28%) = 8% + 7.5% + 14% = 29.5%, 18. A man's wages were decreased by 50%. The reduced wages were increased by 50%, He had a loss of (a) 25% {0} 25% (c) 20% (d) None of these Ans. (b) Here, x = -50 and y = 50. The net % change in wages xy =(x+y+ 4x ( ae S250) 5 os 2D as =(-s0+80- Since the sign is -ve, he has a loss of 25%. 19. The radius ofa spheres increased 10%, The surface area increases by (@) 21% {b) 31% © 41% {@) None of these Ans. (a) Since 4r x radius x radius = surface area Net % change in area y =(x+y+ Xn, ( +) 10x10 100 =(s0+104 21%. oooo 24 |_* Reasoning & Aptitude MADE EASY y Percentage 7. Ifthe side of a square is increased by 30%, its area is increased by (a) 49% (b) 69% (©) 79% (@) None of these 1 Practice Exercise: | 8. In measuring the sides of a rectangle, one side is 1. Ifa number is 20% more than the other, how much Fra the ea excess and te other 20% in dtc pparcentis the second number less than the fist? Fret Qrer percentin area calculated tom the measurement 1 2, (2) 12% deficit (b) 10% deficit oe © 165 (6) 12% excess (a) None of those 1 9. Water tax is increased by 20% but its consumption (©) 165% (d) None ofthese \s decreased by 20%, The increase or decrease in the expenditure is 2. lithe given two numbers are respectively 7% and (a) 4%decrease —_(b) 4% increase 28% ola third number, then what percentage is the (©) B%decrease (a) 8% increase fist ofthe second? 10. The population ofa city increases at the rate of 10% ! (a) 20% (b) 25% annually. Its present population is 90.51 lacs. The {e) 1% (2) None ofthese Population 3 years ago was nearly 3. The price cooking oil has increased by 15%. The (@) 72lacs (0) 68 lacs percentage of reduction that a family should effect, () 80lacs (@) None of these in'he use of Cooking ol so asnotta increase the 14, The value otamachine is Re, 6250, decreases by ‘expenditure on this account is 10% during the first year, 209% during the second 2 1 year and 30% during the third year. What willbe the f@) 195% () 135% value ofthe machine after 3 years? (@)_ Rs. 2660 (0) Rs. 308 ome _ (c) Rs. 3150 (@) None of these 12. An army lust 10% its men in war, 10% of the 4. The difference between a discount of 35% and two Femaining due to diseases and 10% of the rest were successive discounts of 20% and 20% on a certain Gisabled. Thus, the strength was reduced to 729000 4 bill was is, 22. Find the amount ofthe bill active men. Find the original strength (2) Rs. 3200 (b) Rs. 2200 (@) 1000000 (©) 1200000 (c) Rs. 1800 (9) Noneofthese (©) 1500000 (@) None of these 5. Two shopkeepers sella radio ol siniar brand and 43. tn an examination, 30% and 35% students {ype at the same list price of Fs. 1000. The first respectively failed in History and Geography while allows to successive discount of 20% and 10% 27% students failed in both the subjects. if the and the second allows the successive discount of number of students passing the examination 248 2b an tS Find the diference in discounts find the total number of students who appeared in offered by the two shopkeepers. @) Rs. 3.50 (0) Ps. 1.80 the oxaminativn, (c) Rs. 2.50 (A) None ofthese (a) 425 {) 380 (©) 400 (2) None of these 8. The taxona commodity is diminished by 10% and 14. Inaan examination, there were 2000 candidates, oul its consumption increases by 10%, Find the effects onrevenue. (@) 1% increase (0) 3% decrease (b) 2% increase (d) None olthese of which 900 candidates were boys and rest were Girls. If 32% of the boys 38% of the girls passed, then the total percentage of failed candidates is * Percentage | 25 (b) 64.7% its areal. (@) 35.3% 3 (@ 70% :) 68.5% Ifthe price of gold increases by 30%, find by how = much the quantity of omaments must be reduced 10 side fs thatthe expenditure may remain same as before? in deficit > from the 2 ik (275% (&) 275% LE (@ s% (2) 19% +16, The price of an articles cut by 20%. To restore ito sumption} its original price, the new price must be increased creaseint yy © @ 20% (0) 225% © 25% (a) 40% 17. Ina fraction, numerator is increased by 25% and the denominator is diminished by 10%. The new: tte of 10%) lacs. The A fraction obtained is 5. The original fraction is a 5 @s iy reasesby) second: 3 willbethe © & (d) None of these goag 1% of the . atest were Solutions 1072900) 4, Ans.(b) x100}% i 10048 students, aphy while! ( 20 2 = 100} = 165 cts. If the, ¥00+36 © 165% tion is 248, speared in® Ans. (b) | Here, = 7, and m= 28 First number , = —x 100% ofsecond number idates, ou fe 7 aed = 55% 100% of second number Is passed 28 datesis Or 25% of second number, 3. Ans. (b) Reduction in consumption P : 100} (ase ) 5 eee -(aeg*109} % or 1335% 4, Ans. (b) ‘The equivalent discount of two successive ciscounts cof 20% and 20% ~[sve 220) 305 =(-20-20+ Given: 36% - 35% = Rs. 22. Amount of the bill = 22 x 100 = Rs. 2200, 5. Ans. (c) ‘The equivalent discount of two successive discounts of 203% and 10% xy =(s+y+2% loo ( oe) 20 0 =(0-10+ Discount on the list price of radio offered. by the fist shopkeeper 28 = 28% of 1000 1000 = Fop *1000= Fis. 280, Also, the equivalent discount of two successive discounts of 18% and 15% xy =[r+y+2%)oo (5 15x18 3 =(-15-15+ 18218 og or 273 ( a) ) ar Discount on the list price of radio offered by the second shopkeeper. 3 4 = 275% of 1000 = =x 1000 =Rs.277.50 Difference in discounts offered by the two shopkeepers = Rs. 280—-Rs. 277.50 = As. 2.50. 26 | * Reasoning & Aptitude MADE EASY my 6. Ans, (d) Since tax x consumption = revenue Net % change in revenue 11. Ans. (c) Here, A = 6250, x = ~10, y= -20 8 z= 30. 2+ Value of the machine after 3 years i “) ! =[xry+ ele alt al! Sl +) ( ry ( 700)!" F00 Jk 700 (-10+104 012) 7 ~o2s0( 1-2 (1-20 (nere.x = -10 and y= 10) = -1% 700 The revenue decreases by 1% . 7. Ans. (b) = §250%90%80%70 _ a. 3150. " Since side x side = area 100% 100% 100 Net % change inarea 12. Ans. (a) -[oe)s Let Abe the orginal strength 100 Then, Afesds|(t)() aa 706 )\"* 700 )\"* t00. = [so+a0+ 700 = 60% = 729000 (Given) The area is increased by 69%. Ce ae age ale 8. Ans. (a) (2-3-8) Since side, x sidey= area Yo” 100)" 109, = 729000 Error % in area {rye 5 100 hw Z28000% 100 100% 100 10x20 909090 Seopa t a} a (Hore, x= 10and y =-20) 43. Ans. (c) 4 = 12%, Le. 12% deficit. Percentage of students passing the examination” 9. Ans. (a) = (100-(80 + 35 - 27))% Since tax x consumplion = expenditure thee, = 20, y = 95 and» 27] Net % change in expenditure Re amare Lot the foal number of students appearing in tr xv =(r+y+ 2% examinations ( sya) 2. Given :62% of x = 248 20120) = (20-20-2052) )o, 5 - aay =-20) ( 100 Ce ee 210 = 4%, 100 @ Expenditure decreases by 4%, Therefore, 400 students appeared in th examination : 10. Ans. (b) We have, P = 90.51,¢= 10 andn=3 14. Ans. (b) The population 3years ago Boys = 900, Girls = 1100 a Passed = (32 % of 900) + (38% of 1100) r 288 + 418 = 706 toe Failed = 2000-706 = 1294 = 9054, 100, 100 700 “140 "110 1204 Failed % (ee 100}s.=64 1% 1 ination sing in th din the 30 700+30 1 + 100% = 28-7 % New price must be increased by % a x10} = 25% © (00-20 -17.Ans, (a) x Let the fraction be > x40.25x 5 x(1.25) _ y-0.10y~ 9’ (03) Practice Exercise: Il Of the total amount received by Prema, 20% was. Spent on purchases and 5% of the remaining on transportation. If he is left with Rs, 1820, the initial amount was: (a) Rs. 2800 (©) Rs. 2400 (b) Rs. 2000 (d) Re, 1600 The price of jute has been reduced by 20%. ifthe feduced price is Rs. 800 per quintal, the original price per quintal was (@) Rs. 900 (0) Rs. 960 (b) Rs. 640 (d) Rs, 1000 8. 519 part of he population in a village are males. If ' i 30% of the males are married, the percentage of Unmarried females in the total population is: (2) 70% (0) 40% i © 275% {d) 20% A.school has only three classes which contain 40, £0 and 60 students respectively. The pass Percentages of these olasses are 10, 20 and 10 ‘espectively. The pass percentage of the school is: 10. 11 * Percentage | 27 @ 132 © 5 3 2 ©) a (@) 165 If.x% of a is the same as y% of b, then 2% of b is Z%ota i} Zola ©) Foote (@) None of these Froma containerhaving pure milk, 20% is replaced by water and the process is repeated thrice. At the end of the third operation, the milk is: (a) 40% pure (b) 50% pure (c) 51.2% pure (d) 58.8% pure The salaries of A and 8 together amount to Rs. 2000, A spends 95% of his salary and B, 85% of his. If now, their savings are same, what is A’ salary? (@) Rs. 1500 (0) Rs. 1250 (©) Rs. 750 (@) Rs. 1600 300 grams of sugar solution has 40% suger in it How much sugar should be added to make it 50% in the solution? (@) 109ms (0) 40 gms (c) 60 gms (@) 80. gms Inan examination, there are 3 papers of Mathematics of 100 marks each. A boy secures 60% in the first Paper and 70% in the second paper. In order to secure 70% in the aggregate the percentage of ‘marks he should secure in third paper wil be: (a) 90% {b) 80% (©) 75% (@) 70% ‘Two numbers are less than a third number by 30% and 37% respectively. How much percent is the second number less than the first? (@) 10% (b) 7% (©) 4% (3% In an examination, & got 10% marks less than B, B got 25% marks more than C and C got 20% less than D. If got 360 marks out of 500, the percentage of marks obtained by D was (@) 70 (b) 75 (c) 8 (d) & 28 | © Reasoning & Aptitude MADE ERS: ; 12. Ifthe numerator ofa fraction be increased by 15% 19, Ram sells his goods 25% cheaper than Shyam ane andi denominator be dimimished by8%, the value 25% dearer than Bram, How much percentage « of the fraction & The original fraction is a —. heap ee " @ 2 a 2 (©) 06.66% (a) 40% . 5 20. Rajiv wanted to subtract § from a numbe, ©F @ 5 Unfortunately, he added 5 instead of subtracting Find the percentage change in the result. 13. In an examination, 35% candidates failed in one fe) 300% subject and 42% failed in another subject while 15% (0) 66.66% failed in both the subjects. If 2500 candidates (©) b% 1 appeared al the examiniation, how many passed in (a) Cannot be determined either subject but not in both? (a) 225 (o) 1175 21. The salary of Amit is 30% more than that of Varue (©) 2125 Neco Find by what percentages the salary of Varun les than that of Amit? 14, The boys and gils in a college are in the ratio 3: 2. (@) 26.12% (&) 23.07% If 20% of the boys and 25% of the girs are adults, (©) 21.20% (@) None of these o peceniege 7 ree a 22. Ram spends 20% of his monthly income on his hous © 7% ers hold expenditure, 15% of the rest of books, 30%¢ the rest on clothes and saves the rest. On counting 18. The price of sugars increased by 20%. As aresult, he comes to know that he has finaly saved Rs, 952 a family decreases its consumption by 25%. The Find his monthly income. expenditure of the farily on sugar willbe decreased (@) 10000 (b) 16000 by (6) 20000 () None of these a - 23. An ore contains 25% of an alloy that has 90% ira’ Es oe Other than this, in the remaining 75% of the or 16. Abuilding worth Rs. 133, 100s constructed on fand there is no iron, How many kilograms of the ore a ‘worth Rs. 72,900, Atter how many years will the value needed to obtain 60 kg of pure iron? of both be the same if land appreciates at 10% p.a. (a) 250kg (b) 275 kg and building depreciates at 10% p.a.? {c) 300kg (2) 286.68 kg (@) 25 (b) 2 24, Ram sells his goods 20% cheaper than Bobby at () 15 3 20 dearer than Chandilya. Haw much percentage 17. A reduction of 21% in the price of wheat enables a Chandiiya’s goods cheaper/dearer than Bobby's. person to buy 10.8 kg more for Rs. 100. What is the (@) 33.33% (©) 50% (©) 42.85% (@) None of these reduced price per ka? (@) Rs.2 (b) Rs. 2.25 25, Outof the total production of iron from hematite, & (©) Rs. 2.90 (a) Rs. 250 ‘ore of iron, 20% of the ore gets wasted, and outt the remaining ore, only 26% is pure iron. Ifthe put 18. The length of a rectangle is increased by 60%. By iron obtained in a year from a mine of hematite we what percent would the width have tobe decreased £80,000 ka, then the quantity of hematite rmined ita” to maintain the same ara? that mine in the yearis 4 (a) 5,00,000kg —_(b) 4,00,000kg @) 375% (b) 60% (¢) 4,50,000kg ——_(d) Nonoof these (©) 75 % (d) None. 26. Ram spends 30% of his salary on house rent, 3 of the rest he spends on his children’s educatt -am and and 24% of the rest salary he spends on clothes. ntage i terhis expenditure, he is left with Rs. 2500. What g Ram's salary? (@ Ps.6713.2 {b) Rs. 20,000 (@) Rs. 10,000 {d) Rs. 15,000 vumbe od tracting Solutions 4. Ans. (b) - Purchases = 20% of == his hous 8, 30%¢ counting os Fis. 9528 Bo. 12EL 1500 <5 x= 152028 - 2000, | 19 © 2. Ans. (d) 90% iro, 80 am of = 800 3 athe of 800 = 5g" = 800 reoreat 40% = r= 800x102 - 4 0 7 1000 3, Ans. (c) Let, total ion = x, a ao al population =. Males = 2 centagel, Married males = 90% of Bobby8;§ 305 1 9*"700%9*"6 omatite,, | Married females = and out! Ifthe pt Total fe ae tal males ={ ) nined fr 6 Unmarried females = (é : Required percentage Sed 7 >rent, ( x hitco) 272 educat 180 x 9 * Percentage | 29 . Ans. (a) Number of passed candidates 10 20 10 =(22 x41) “x60 fe 402x604 x00] = (4 410+6)=20 20 Passed percentage 0 x (aors0+60) *' 20 1 =x te = 13%. & 100} 3” . Ans. (c) x% of a= (oF Ans. (c) Let total quantity of original milk=1000 gm. Milk after first operation = 80% of 1000 = 800 gm. Milk after second operation = 80% of 800 = 640 gm. Milk after third operation ‘Strength of final mixture = 51.2%. . Ans. (a) Let A’s salary = x, Then, B's = (2000 - x) 5% of A =15% of B, i.€. 5, 8, ay (2000-x) or x= 1500. 00" . Ans. (c) Sugar = (% x0 ns =120 gms, water =180 gms, Let.x gm sugar may be added. 120+x 100 = 50=> 240+ 2x Then, 903% + =300+1= x=60 30 | * Reasoning & Aptitude 9, Ans. (b) 60+70+2= 22 300 orx = 80% 100 10. Ans. (a) Let, third number be x. Then, Tx 14, Ans. (c) Suppose boys = 3x and girls = 2x. 80. 75 Not adults = (=>. aL jot adults. (B2}-(Bx2s) (& %) 39x eto) € First number = 70% of x = o” 52 10 a Required percentage 2 ‘Second number =63% of x = ——. {00 5 (F xx too Required Percentage 10 Sx =(Bxfo 100} =10% 15, Ans, (a) 100 7x Let original consumption=100 units & original prici 16 = Rs. 100/unit, ee Original expenaiture = Rs. (100 x 100) 90 125 80 = Rs. 10000. A= Fog Be yag © an8 mig? New expenditure= Rs, (120 x 75) = Rs. 9000, Decrease in expenditure rod 5, Rac~ Sa anap = 2c -( 2) “om 10 4 (0000 x 360 = 400, C aoe 8 16. Ans. (d) 5 and D => x320= 400, " n 20 : r2900{ 1+ 0) =133100%(1-72) 400 100, 400, Percentage of D= (an *100)* = 80%, . (2) 40)" _ 133100 _ 1331 12. Ans. (b) “0, “72200 ~ 729 24 Let the given fraction be x/y. 115% of x , (y then, L1S%otx . hen. 5% of y 9 17. Ans. (a) Let original price = Rs. x/kg. Reduced price 79 22 =| ——x |/k 13. Ans. (b) (& a 7 35. Failed int* subject = (& x 2s00} 875. 400 100 gg , 19000 100196 oe ones 7x x Failedin 2 subjoct= (3 x 2800) = 150 100 10000 - 7900 = 10.5 x 79x 15, Failed in both (20m) -078 2100, 100 oo 10.5x79 Failed in 1® subject only =(875 ~375) = 500. “Reduced price Failed in 24 subject on! 1050 - 375) = 675. on . Passed in 2° only + Passed in 1*only . (op 00 ys =Rs 2/kg = (675 » 600) = 1175. 05x79 . (a) 410000 460 125) 471 P x=(100-3) 375 ‘ginal pio r 100—x = 1. ‘ns. (d) Let Shyam sells good at Rs. 100 So Ram sells good at Rs. 75 Now this Rs, 75 is 25% dearer than Bram ie. 9000 | Bram price = 60 ~ Bram’s good is F's. 40 cheaper than Shyam in % 40 term 2 9 100i. 40% a Ans. (d) | This cannot be determined because after adding or | _substracting So attferent numbers variable results | _wecbizne. ‘21. Ans. (b) © Let Varun's salary is 100 | Amitsalary =0% more that Varun's Salary = 120in 1% salary of Varun less than Amit a) = 2 x10 = 23.07% price § 130 2. Ans. (0) Lethe eams Rs, P 5-20, y=-18, 25-30 05 * Pl sy [ mall pl wl 80], f 85 If 70 Px] = LOT [is lt Fal $20 Ps. 476 = 9520, P = 20000 kg * Percentage | 31 28, Ans. (d) Lt amount of ore is 100 kg 25% oF 25 kg has 90% iron _ 25x90 “100 75% oF 75 kg has no iron = 0 kg To obtain 22.5 kg 100 kg ore is required So to obtain 60 kg 100 Bs 2.5 kg iran sex 60 = 266.66 kg 24, Ans. (a) Let Bobby's sale price is Rs. 100 Ram sale price = Rs. 80 Ram sale price is 20% dearer than chandilya So, Chandilya's price = CP 120 aa~ = 80, CP = 66.66 OP x555 = 80 Now Chandllya good is Rs. 33.83 cheaper than Bobby's ie. = B88 5 100 «33.39% 25. Ans. (b) Let kg ore is there 20% washed away soremaining is 80% i.e, 4/5x 1 24 out of 2 én 25% is pureiton Sox Er = Ee 4 ile. §* kg Is obtained from x kg then 1 kg is obtained from 5 kg 80000 kg is obtained from 5x 80000 = 400000 kg 26. Ans. (a) Let total salary is Rs. x P| 2 5 1 100. 100. [Give sign because of spending} 70 }f 70] 76 li [Tahoe 2500 P=Rs. 6713.21 eee iat Inany organisation, the most important aspect is profit and loss calculation of any transaction conducted. Su the profitfoss calculation is having @ very important impact from business point of view. While dealing with the proiitfioss calculation, the following terminologies have to be understood, ‘Terminalogies in Profit & Loss Cost price: Cost Price (CP) of an atticle is the ‘expenditure incurred on the purchase or the duction of the article. I can be taken as the "INVESTMENT on the article. Selling price: Selling price of an article is the revenue generated by seling that particular article. Soitis the REVENUE’ Both CP & SP are respective quantities. They are not-absolute-quaniitias Lethe value keeps on changing with respect to the person. If Ais selling a quantity to B. In that case selling price for A is the cost price for B. Profit/Loss: The difference of SP & CP is the profit value or oss value. If SP > CP, the value is profit. SP < CP, the value is loss. Percent Profit/Loss: The profit or loss percentage of the ‘INVESTMENT is profit or percentage. Profivoss Value % Profitoss = — ——— Investment Margin: Margin is generally used in terms of percentage. Itis the profitor loss as percentage of the ‘REVENUE or selling price. ProfifLoss value , Revenue Marked Price: It is the price of the product being displayed on the label. Generally tis taken as MAP. 100 100 % Margin = Discount: Discount is the rebate given to the customer before the seling of the article. Generally discounts given on marked price but to havea loss the discount can be givan on the cost price 2s well Profit and Loss Mark up: This is the increment on the cost price before the atticleis sold to the customer, From these terminologies it has to be understooc that the relation of selling price and cost price it nothing buta percentage increment or decremer E Situation based on profit or loss respectively. So profitorlossisP% we can use themutiplying factc ie ut] & (709 P So SP = oP( 1 8 0 ( =) 7 Now let us understand it with the help of som examples, Example 1 Ram bought @ scooter for Rs. 10000 and sold it 20% loss to Rahul. Rahul invested Re. $00 in aT SOTAT TOs BrOMTe Soha Fincoutthe armen paid by Sohan, Solution The CP for Rahul 20 8 10000] 1- ) Se ( 100. = Rs, 8000 Net investment of Rahul on the scooter = 8000 + 500 = Rs, 8500 Rahul soldi for 10% profit So amount paid by Sohan 10 = 8600 1+ oo + i) Example 2 Komal sold two articles at same price & ‘at 20% profit while another at 20% loss. Find his net prof or loss percent? 's, 9350 Solution. Let the SP of one article be x One article is sold at 20% profit so 12x(CP), = x @, larly 08 x (CP), So loss 2x(0.04)/0.96 2x(0.04)/0.96 oe 2x/0.96 = 4% loss Rose bought a bullock and a cart in rupees 6000. ‘She sold bullock, at 20% profit and cart at 10% tion. et the CP of bullock be ‘b’ & of cart be ‘c’ Then b+ = 6000 and 1.2b4+1.1¢ = 7000 {By solving both the equation we get sold tte b = 4000 and c = 2000 5.500 ing SoCPofbullockis Ris. 4000. MOU hea) Example 4, Ashopkeeper gives a discount of 10% on a article but his net profit becomes 8% still. Find out by what percent he mark up the article. Solution, Let the CP be x & the mark up be P% 8 Then SP = (: a |x t + 00 (As per the profit) P 10 sp=(1+2),()-42 ( + a) ( 10.) As per markup (Since mark up& discounts are successive percent 1350 changes on CP) 8 (112 oor=s.08. 700. J0.92 = 1.08: ng price ass. Find. Ise Weight ‘| Sometimes the shopkeeper keeps the CP & SP of the articles same, But instead of giving us the neem grees eet 4 a). si » Profit and Loss | 33 indicated quantity he supplies less quantity & makes profit, So for the caloulation of profit percentage we have to see his investment & the. investment is the quantity he is providing to the ‘customer. So profitar loss percent is calculated on the amount being supplied to the custorner. Example 1 A shopkeeper sells the quantity in the same price rate for which he has bought. But he gives 20% Jess quantity to the customer. Find his profit percent. Solution Let the indicated weight be 100 gm & CP & SP be 1 RsJgm Now quantity given to the customer = (1-02) x 100 So investment of shopkeeper = 80x 1=Rs. 80 ‘The amount gained from the customer = 100x 1= Rs. 100 100-80 Profit % = 100 = 25% profit 8 Example 2. Ramesh purchased a radioset at Rs. 1500 and soldit at Rs, 1200. Find loss incurred by him? Loss= CP - SP = 1500-1200 = 300. Also in this case we can calculate 300 20% Thus he incurred 20% loss. Basic Formulae 1. When SP and Gain% are Given then 100 9 =(— 10 _),sp, = (arto) 2. When the CP. and Gain % are given then. 100+Gain% 100) SP. cp. 3. When C.P. and loss% are given then 100 Loss xP. 34 | © Reasoning & Aptit 4. When SP. and loss percentages are given 100. ) 100 C2 (ar tam 5. Ifthe cost price (C.P. of articles is equal to selling price of natticle, then % gain or loss = [PE] «100 lf m>n, itis % gain and if m 70% Ex.2__ Finda single discount which is equivaient toth successive discounts of 20%, 30% and 20% Sol.: Single discount equivalent to 20% and 30% = [2020-225 100 = u% Now, we will find single discount whict equivalent to two successive discounts of + and 20%. 44x20 44420-. . [ +20- Se i > 55.2% Alternative: xx O.8r x 7x x 8 = 0.448 So the final value is 0.448x Which is reduced by 55.2% Sooo * Profit and Loss | 35 dentto thie g, and 20% and 30% i i at which unts of perce cape (@) 625% (b) Rs. 650 (@) None af these SP. shopkeeper sold goodss for Rs. 2400 and made a profitof 25% in the process. Findhis profit percent he had sold his goods for Rs. 2040 (b) 7% © 6.20% (0) 6.5% Ans. (a) SP = 2400, Profit% = 25 se 2400 P= P5199 = 208 109 = OP agepagy 100 = S100 = 1020 If sold at 2040, profit = Fis. 120 120 Profit % => => 100 = oft % = <5 100 = 6.25 A digital diary is sold for Rs. 935 at a profit of 10%. ‘What would have been the actual profit or loss on it, ifit had been soldi for Rs. 810? (@) Ps. 45 (b) Rs. 40 (6) Rs. 48, (a) Rs. 60 Ans. (b) SP = 935, Profit %= 10% 935 cp = 8 509 = 779% 100 = 850 Diary if sold at 810, Loss = As. 40 A shopkeeper bought 240 chocolates at Rs. 9 per dozen. the sold all of them at As. 1 each, what Was his profit percent? (a) 66(116)% (o) 33(1/3)% (ce) 24% (d) 27% Ans. (b) OP of 12 chocolate = Rs. 9 CP of 1 chocolate = 2 = Rs.0.75 Now SP = Rs. 1 Proit = 0.25 1 it % = 2x 100= 332% Profit % = Gae* 35 8. 0.25, . A coal merchant makes a profit of 20% by selling coal at Rs. 25 per quintal. Ifhe sells the coal at Rs. 22.50 per quintal, what is his profit percent on the whole investment? (@) 6% (b) 6.66%. (©) 7.5% (d) 8% Ans. (d) Profit % = 20%, SP=25 cp= 28 x100= As =20.83 720 6 Profit if SP = 22.50 = 22,50 ~ 20.83 = 1.867 1.667 20.83 [in fractional term this can be solved very easily} Profit % «100 = 8% |. The cost price of a shift and trouser is Rs. 371. If the shirt costs 12% more than the trousers, find the cost price of the trouser. (a) Rs. 125 (b) Rs. 150 fo) Rs. 175 (@) Rs. 200 Ans. (c) Let CP of trouser be Rs. x 12x = As Now CP of shirt = P85 Vax _ According to given condition x+ Foe = 371 212 212 37, 100" ~° x= Rs. 175 . The marked price of a table is Rs. 1200, which is 20% above the cost price. Itis sold ala discount of 10% on the marked price, Find the profit percent (@) 10% (©) 7.5% (b) 8% (6) 6% 36 | * Reasoning & Aptitude MADE ERS yy If sold at 10% discount (a) 2307 (0) 20% 43 20 (0) 21.21% (a) 25% SP. = 200% = 1080 Ans. (b) Profiton GP = Rs. 80 Lot the cost price be Rs. x and marked price bes.) Profit %6 = G= 36, P=36x2=72—P = 24 30 = 24x 2= 48 => C= 16 Cost of a cricket ball=Rs. 16. 2. Ans. (c) 90: 10.80 Hence, S.P. = Rs, 14.40 40 | © Reasoning & Aptitude MADE EAS; 3. Ans. (d) Required ratio = 30: 20 = 3 : 2 15 Least CP. = Rs. (200 x 8)= Rs. 1600. Greatest $P. = Rs, (425 x 8) = Rs. 3400. os ) Required profit = Rs. (3400-1600) Gain % (23x00) = 11d, = Rs, 1800, 900 9 4. Ans. (c) 40. Ans. (a) Let C.P. of each article be Rs. 1. CP. of 1Barticles = Rs. 18 er SP. of 1Batticles S.P.of 3 toffoes = 150% of Re 1 = Rs.5 3 2, i fa * 100% = 165% For Rs. 3 toffees sold = 3. 16 5. Ans. (c) 2 Gain = (SP. of 250 chairs) - (C.P. of 250, For Rs. 1, toffees sold = (2 x 3) =2. chairs) . (SP of 250 chairs) - (CP. of 250 chairs) 11. Ans. (o) = SP.0f 80 chairs. ‘S.P. of 200 chairs = C.P, of 250 chairs. Let S.P. of 45 lemons be Rs. x Let CP ofeach chair be Rs. 1 a0 _ 120 CAP. of 200 chairs = Rs, 200, ae S.P. of 200 chairs = As. 250. 40%120 50 o a 7% ae (2 j 100} oan For Rs. 60, lemons sold = 45, For Rs. 24, q7 6. Ans. (c) ‘Suppose, the number of books purchased is 110 ere oe (Bees) 60 10. CP. of 110 books =Re{ M110] 42. Ans. (b) San 110 x= (100 +P): oF Pat 10th = (794710) x nose or 100+P = 220 = Rs. 121, Profit % = 21% p =120%. 7. Ans.(d) 43. Ans. (b) CP. of 25kg = Rs. (15 x 14.80 + 10x 13) 125% of 120% of A= 1500 = Rs. 347.50 S.P of 25kg = Rs. (25 x 18) = Rs. 375. 2 Ba. 1500 Gain = Rs. (375 ~ 347.50) = Rs. 27.50 18 8. Ans. (b) Mean price =Rs( 122.06 |= Rs. 20m lean price = Re} 555% 96 |= 8 14. Ans. (b) By the rue of alligation: CR oft te Ghee CP oft kg a 7 400 “700 Mean Price 253, a or FE p= 1266, E é (1265x160 : a0 % pl ) Rs. 800 © Profit and Loss | 41 (c) 19. futplying 2° equation by 2and subtracting first fiom it, we get 19600 ~ 7000 =12600 or y = 4200 C.P. of aT. V. = Rs, 4200 Ans. (b) Total S.P. = Rs, 24000 CP. of horse nile panes cnn *. ) ona is. 25000. 4000 - 25000 = Rs. 1000 Ans. (c) Let original SP. be Res. x, News.P.=2,, loss = 10% 24. 8. Ans. (¢) Letus consider a packet of joe marked 1 kg, Hts actual weight is 80% of 1000 gm 22 = 800 gm, LOtCP. of each gm be Rs. 1. Then, C. P. of this packet = Rs. 800. SP of this packet = 110% of CP. of 1 ka, te a ={spqr0}- 1 300 Gain = (200 (S100) 7.5% Ans. (b) Let the total value be Rs. x Value of Sra 3, Value oftrd= 3 Total SP. =( 105% of 24-98% of otal (102% ot 2-963 of3 (ees ar) 308x 5 eae ‘300 * 300)" 300 308x 308x ~ 300x = x2 400 = EEA S008 Ly 300 300 a 300% 400 = 16000 Ans. (c) ‘Suppose, the quantity sold ata loss be.x kg and let CP. per kg be Rs. 1 Total C. P. = Rs, 24. Total S. P. = 120% of (24 — x) + 95% of x. 19x _ 576—24x+19x 20 20 _ 576-5x 20 576~5x = 110% of 24 S76-5x _ 264 20 10 of S76-Sx=528 oF Sx= 48 of x =9.6k9 Ans. (a) 40 5 cp.- 2 xsp. 709 “SP. 18. SP.=3CP. ~(Zar00}s ocr 2 SP. = 250% of CP. Ans. (a) Let CP. be Ris. (105% of x) ~ (95% of x) = 6.72 or 10% of r = 6.72. Ans. (c) Let original C.P be Rs. x/lts poeeens 100" 20 eon aias New cp=25. ow CP ao5* = 39 42. | © Reasoning & Aptitude MADE EASY er = {100-3 peor 315 -( Basen) 121.875 24. Ans. (6) Total CP. = Fis, (12x 4 +16 x 2) = Rs. 80. Profit = 21 o75= ee SP. of 6 dozen oranges 29. =Fa( Fogo % 20% Sir Fis.183.60 a ‘S.P. per dozen = Rs. 16 ‘ort 25. Ans. (b) Let, the marked price be Rs. x The Let CP. be Rs. x i ate 900 -x = 2 (x- 450) yoo" "182.60 => 3x =1800 = x = 600. ee 83.60 100 _ og CP. = As. 600, gain required = 25%. 8 sp. = R(T" 600) Fs 750 30. Ans. (b) 00 Lt, the list price be Ris. x Co 26. Ans. (b) 8 og my y= 24%100 _ 59 te Let CP. be Rs. x, 100° 80, orn 2-75) = (96-2) Required SP. = 70% of Rs.90 = Rs. 21 Bx = 24 _ 384. Ans. (b) _ Let the CP. be Rs. 100. = Then, SP. = Rs. 112. 27. Ans. (d) Let the printed price be Rs. x Cost price = 95% of 90% of 80% of Ris. 300. es Dae eta rete 95 , 90, 20 Then, 90% of x= 12 755 ae = 00" 700" 700" FS 0520 (He) 1120 28. Ans. (c) 90 9 -- Let C.P. be Rs. 100. Then, marked price 4120 ~ = As, 130. (CP): (Printed price)= 100:5° += 900: 1120 = 45: 56 Jar Sat fre sav 0 ‘Benalla Je interest ofthe previous years are added tothe rrincipal for the calculation of the compound interest. Ry b Cl=Pit+t] - i [1 a Pp [Where P is principal amount t Ris Rate of interest Tis time duration 22861: When interest is compounded annually hen Jount Awillbe ' iT | A-[u 100. fase 2: When interestis compounded hal yearly then wa A ‘r [1 el ise 4: When differentia rate of interest is charged i. fale of interest is | Ror ist year i R,% for second year and R,% for third year then [estal-L itl Amount =P} 14 Qoa0 Simple Interest & Compound Interest Example 1. ‘A sum of 12000 Rs. is deposited into SBI Bank for 3 years. If the bank is providing 6% rate then culate the amount after the maturity period Solution gi= PRE 700 1 = 120005%3 _ps 1200 100 ‘Amount = Principal + SI 19800 = 12000 + 1800 Example 2. ‘An amount become double in years calculate the e of interest Solution: Let the amount be Rs. « ce it become double in 8 years so SI in8 years, =Rs.x Example 3. ‘An amount become 5 times in 20 years at simple interest, Calculate the rate of interest given Solution: Principal = Rs. x Total Amount = Rs. Sx. So Sl = Rs. 4x PR xxix 20 Now gi= PRT, gy = 1*8x20 OW S = 05 100 44 | * Reasoning & Aptitude Example: 4. ‘An amount of 25000 is deposited in to Citibank for 2 years. Calculate the interest incurred if rate of interests 10% compounded annually. Solution: Ans. (a) Here P = Rs. 2000, R = 5%, T ae Rs. 26000 : {fees 18, i a R= 2years i ay 200% 5%2 7. Fin ciple a -P Wy rat 100. = 205 - 200 = Rs. 5 f - 2soen| 10" T 25000 Alternative: 7 © 100. Difference between Cland SIfortwoyears =f AM = 30250-25000 = Rs. 5250 ‘ Rel Sy agoa (or-si= *(-R5) =@0( 55) <8 | 4, Whatis the difference between the simple inter Solved Example: 2 ‘ona principal ofRs. 500 being calculated at 5% annum for 3 years and 4% per annum for 4 ye 4. Rs, 1200 is lentout at 5% per annum simple interest (@) Rs.5 (b) Rs. 10 . for 3 years. Find the amount after 3 years. (c) Rs. 20 (d) Rs. 40 ‘ (a) Rs, 1380 (b) Rs, 1280 Ans. (a) 5 (©) Rs.1470 (a) Rs. 1200 is. 500, R, = 5%, R= 4%, : Ans. (3) years, T,=4years 8. Here P = Rs, 1200 R = 5%, T= 3 years Difference = 007 100-7 Rs. 5. ¥ 2 A _PRT _ 1200x5x3 © 100 100 Amount = P + Si = 1200 + 180 = 1380 si = Rs. 180 Rs. 2100 is lent at compound interest of 5% per annum for 2 years, Find the amount after two years, (@) Rs.2300 — (b) Rs. 2315.25, (o) Rs. 2310 (d) None of these Ans. (b) Here P = 2100, R = 5%, T = 2 years. Ry sf Plisce) 2,00) 1+. [il 209| t+; 21, 24 = 2100x2121 - Rs 2915.28 ae Find the difference between the simple and the compound interest at 5% per annum for 2 years on a principal of Rs. 2000. (@) 5 (b) 105 © 45 (6) None of these 5. What is the simple interest for five years on asi (¢} of Rs, 700 if the rate of interest for the first yee An Is 8% per annum and far anather ? years is 7%F annum? (a) 400 (b) 392 (@) 352 (6) 266 i Ans. (d) i 700x3x8 700x7%x2 700 oo 78 Aig 6. Find the compound interest on is. 1000 at them cof 20% per annum for 18 months when intere# compounded half-yearly. The (@) Rs. 331 (b) Rs. 1331 He (c) Rs. 320 (dq) None ofthese la) Ans. (a) © Here P = Rs, 1000, R = 20%, A 1 T= 18 months or 15 years. = 001 5] 3 roo] = Rs, 1931 the principal i the interest compounded at the cof 10% per annum for two years is Rs. 420. F (©) Rs, 2200 (d) Rs. 1100 ; 20100 es =Rs.20 ‘ a ee }. What's the rate of simple interest for the frst 4 years [ifthe sum of Rs, 360 becornes Rs. 540 in 9 years ~and the rate of interest for the last 5 years is 6%? ~ @) 4% (©) 3% Ans. (b) P= Res, 360, A= Rs, 540, SI =Rs. 180. PRT 00° (b) 5% ‘gona (d) 6% first 3 sis 7% i sl 190 = 350XR%4 | 360x6%5 100 100 72.= SORA 5% > Alay borrows Rs. 1500 from twomoney-lenders, He Pays interest at the rate of 12% per annum for one f loan and atthe rate of 14% per annum for the other. The totalinteresthe pays forthe entre yearis Rs. 186. How much does he borrow atthe rate of 12%? Oatthe n inter (@) Rs. 1200 (b) Rs. 1300 (©) Rs. 1400 (d) Rs. 300 Ans. (a) P= 1800,R, = 12%, R= 14%, T= 1 year SI= 186 * Simple Interest & Compound Interest | 45 Let Rs. x be amount invested at 12% Rate elt (1500=s) «14 1 100 100 ee 21000 2x SO 2 2186, x= 70077007 88 += 1200 10. Two equal sums were borrowed at 8% simple interest "1 12 per annum for 2 years and 3 years respectively. The difference on the interest was Rs, 56. The sum borrowed were (@ Rs.690 (0) Rs. 700 (©) Rs. 740 (@) Rs. 780 ‘Ans. (b) Eee Eaay 700 ~ 700 8x3-8x2]_ 5. 5 [sare =56, P= 700 If the difference between the simple interest and compound interest on some principal amount at 20% er annum for 3 years in Rs, 48, then the principle amount must be (a) Rs. 850 (b) Rs. 500 (©) Ris. 375 (d) Rs. 400 Ans. (c) Here P = ?, R= 20%, Difference Rs. 48 ry 4-0 t+] ~P ‘On solving P = 375, Raju lent Rs. 400 to Ajay for 2 years, and Rs. 100 to Manoj for 4 years and received together from both Rs. 60 as interest. Find the rate of interest, simple interest being calculated (a) 5% (b) 6% (©) 8% (0) 9% Ans. (a) PRT, | PaRoTp PRT FeRele «gy 100 ~~ 100 400x2%R 100%4xR i009 * ig 12R = 60, R= 5% 46 | © Reasoning & Aptitude MADE EASE 13. Asumofmoney doubies itsetfin 5 years, In how many years wilitbecome fourfold (finterestis campounded)? @ 6 (b) 10 © 2 @) 12 Ans. (b) Let sur = [: +A] oe o 14. A sum of money placed at compound interest, doubles itseif in 3 years. In how many years will it ‘amount to & times itself? (@) Rs. 4000, Rs, 2000 (0) Rs. 5000, Rs. 1000 (6) Rs. 3000, Rs. 3000 (d) None of these Ans. (a) Let one part be Rs. then xx2x6 _(6000-x)x3x8 00 gee 100 eee 144000 - 24x 144000 4000 19. 16. A sum of money becomes 7/4 of itself in 6 years, a certain rate of simple interest, Find the rate interest. (a) 12% (b) 125% (o) 8% () 14% Ans. (b) Let sum be x 3 A= x, So, Slndx T=Gyears, R=? (a) Syears (b) 8 years (©) 27 years (@) 7 years SIx100 _ 32x10 Ans. (2) a Let sum= x rO} T = 3 : T=9yrs. Alternative: If certain sum of money becomes ‘m' times in ‘y' years. Then it will become (rm")times in 'n x y' years. Hence 2°in3 x 3 =9 years. 16. Divided Rs. 6000 into two parts so that simple interest on the first part for 2 years at 6% p.a, may be equal to the simple interest on the second part for 3 years at 8% p.a. 17. Sanjay borrowed is. 900 at 4% p.a. and Rs, 11 at 5% p.a. for the same duration. |e had to 91 Fs. 964 nallas interest. Whatis the time periodin yea 1 (@) 5years (0) years (c) 2years (@) 4years Ans. (d) PART, PRpT2 yoo * 100 = 964 [Seg Hows) 400 100 36 +55T = 364 : T=4 years 8 18. Ifacertain sum of money becomes double at sit interest in 12 years, what would be the rate ofinto per annum? i (@) at (b) 10 ( 12 () 14 prebans magc mApE ERSY © Simple Interest & Compound Interest | 47 ‘Ans. (a) (@) Rs.1520 (6) Rs. 1880 xxix (©) Rs.2120 (a) None yee ce 5. The simple interest on a certain sum of money at ay the rate of 5% p.e. for 8 years in Rs. 840, At what 7 rate of interest the same amount of interest can be 49. Asum of Fis, 600 amounts to Rs, 720 in 4 years at received on the same sum after 5 years? simple Interest. What will it amount to if the rate of (@ 6% (b) 8% interest is increased by 2%? (0) 9% (@) 10% (@) Fs.648 ——(b) As. 768 ives | Gases 6. A sum of money was lent at simple interest at 11% Ans. (b) 1 1 P = Ps, 600, Pea p.a.for 3 years and 4 years respectively the Ue Bet difference in interests for wo period was Rs. 412.50, p= SLX100 _ 120x100 _ gy the sum is: aren ONS (2) Rs.3250 —(b) Rs. 3600 (©) Fs.3750 (a) Rs. 4250 g1 = B00%7x4 _ seg 700 7. Prabhat took @ certain amount as @ loan from a bank = 600 + 168 = 768 atthe rate of 8% pa. SI. and gave the same amount to Ashish as a loan al the rate of 12% pa fat the Leola) end of 12 years, he made a profit of Rs. 320 in the deal, wnat was the original amount . (@) Rs.2000 —(b) Rs, 3000 Simple Interest (©) Rs.4000 —_(@) None of these / 8. Rahul borrowed Rs, 830 from Mr. Lal at 12% pa erates rool: Sil. for 3 years. He then added some more money tothe borrowed sum and lent it to Shobha for the H. Avinash borrowed Rs. 5000 from Sanjay at simple same period at 14% p.a. rate of interest. f Rahul Interest, After 3years, Sanjay got As. 300 more than gains Rs. 93.90 in the whole transaction, how much what he had given to Avinash. Whatwas the rate of money did he add from his side? interest per annum? (@) Rs, 35 (b) Rs, 65 (a) 2% (b) 5% (c) Rs. 80 (d) Rs. 105 () 8% (a) 10% 9. Wr, Roopchand finds that an increase in the rate of Rs, 800 amounts to Fis. 920 in 3 years at simple interest. if the interest rate is increased by 3%, it interest from 429410 519% per annum increases would amount io how much? Cas (a) Rs. 992 (b) Rs, 1056 his yearly income by Rs. 25, His investment is: (© Rs.1112 (a) Rs. 1182 a aesoccon aortic cee . The simple interest on a sum of money at 5% is (0) Rs. 15,000 (d) Rs. 20,000 Bs. 48 for 4 years, The simple itereston he same 49, Theale at whicha sum becomes four times ofits sum for 5 years at 4% will be. (@) Rs. 40 (©) Rs. 50 (b) Rs. 48. (d) Rs. 60 A sum of money at simple interest amounts to Rs, 2240 in 2 years and to Rs. 2600 in 5 years What is the principal amount? in 15 years at S.I, will be: (@) 15% ) 174% (©) 20% (d) 25% 48 | * Reasoning & Aptitude MADE ERs: 11. The simple interest on a sum of money at the end of 1 four years is = thof its principal. What is the ate of interest per annum? (a) 4% (b) 5% (0) 6% (A) Data inadequate 12, A sum of money triples itself in 16 years 6 months. Inhow many years would it double itself? (@) 6 years 3 months (b) Tyears 9 months (©) Byears 3 months (d) years 6 months 13, A sum of Rs.10 is lent to be returned in 11 monthly installments of Rs. 1 each, interest being simple. The rate of interest is (b) 10% 1 @ 95 9 = % (©) 11% (oh) 255 14, The rate of simple interest on a sum of money is 6% pa. for the first 3 years, 8% p.a. for the next 5 years, and 10% p.a. for the period beyond 8 years. Ifthe get the same amount after 2, 3 and 4 year respectively. If the rate of simple interest is 5% then the ratio of amounts invested among, A, Ban C willbe: (@) 10:15:20 (© 6:4:3 (b) 22:23:24 @) 2:3:4 ooo Solutions 1. Ans. (a) 100300 Rate = ( 100% 300 og _ 25, ise (Sass) 2. Ans. (a) Principal = Rs. 800, Sl, = Rs. 120, Time = 3 years 4100x120 = | 94 = 5%. g00x3 J Rate = ( New rate = 8%, Principal = Rs. 800 simple interest accrued by the sum for a total period Tene 9 years of 10 years is Rs. 1860, what is the sum? (800x8x3) Shak = R19: (@ Rs. 1500 (b) Rs. 2000 (100, (c) Rs.3000 (d) Datainadequate 18. A monthly installment of Rs. 180 is required to be paid for repayment of an interest free loan in 40 months. If it is decided to pay it in 30 months, how much will be the monthly instaliment in rupees? @ 6 (b) 198 (c) 240 (d) 330 16. A sum of Rs. 1550 is lent out into two parts, one at 8% and another one at 6%. Ifthe total annual income is Rs. 106, the money lent at 8% is: (@) Rs. 650 (b) Rs. 720 (©) Rs. 840 (d) Fis. 900 17. Asum of Rs. 1550 was lent partly at 5% and partly at 8% p.a. simple interest. The total interest received after 3 years was Rs. 300. The ratio-of the money lent at 5% to that lent at 8%. (a) 8:5 (b) 5:8 (9 31:6 (6) 16:15 18. Aman invests an amount of Rs. 15860 in the names of his three sons A, B and C in such a way thatthey New amount = Rs. (800 + 192)= Rs. 992 3. Ans. (b) Principal =o 1-3 s.240. 3x4 pre 240%5%4)_ si Fe ( Sy aR 8 4, Ans. (d) S11 for = Rs, (2600 ~ 2240) = Ris. 360. 60, SL. For 2 years = Rs. (2x2) = Rs.240 Principal = Rs, (2240~ 240) = Rs. 2000 5. Ans. (b) (100x840 sum-{ 5x8 } Rate requires MADE EASY * Simple Interest & Compound interest | 49 6. Ans. (c) Let the sum be Rs. x. Then, Let the original amounts be Rs. x. then, 12x12 _xx8x12_ yoy 700 100) x= 2000 «as 66667 8. Ans. (d) (890+ x)x14«3 _ 830x12x3 100) 100 OF 830 x 42 + 421 — 830 x 36 = 9390 42x = 9390 - 4980 93.90 _4at0 -4- Money added = Rs, 106. 9. Ans. (a) Let the investment be Rs. x, Then, ad 991 A gg IKE GEG = 25 = 2x=20000 = x= 10000. 10. Ans. (c) Let sum = , Then, S.1, = 3x 00x 3x Rate e201 11. Ans, (b) Let sum=, Then, Sh= Time = 4 years at (1002S }o= 5% So xx4, 12. Ans. (b) Let sum = x, 1 Then, Si.=2°8 Time = $year Rate ( x: a WO0x2x 2 400) 2g = (40 7 a)(5) 400 Now, sum = x, S.l= x & Rate (x eee St) ears ¥ “ao0)% = 7 years 9 months 13. Ans. (d) Rs. 10 + 8.1, on Rs, 10 for 11 months = Rs.11 +S. 1. on Ret for (1+2+3+44 + 10) months Rs. 10 +S. on Rs. 1. for 110 months = Rs. 11 +S. on Rs. 1 for 55 months S.l. on Rs. 1 for 5 months = Rs. 1 40x12 6 = 155) } Tine=( 9 Rate «( 14, Ans. (b) Let the sum be Rs. x. Then 15. Ans. (c) 180 x 40 = xx 30 or x = 240 16. Ans. (a) Let the money lent at 8% be Rs. x. then, 2xBx )x6x1 = 106 100 700 of 2x + 9300 =10600 or x = 650, 17. Ans. (d) Let the sum at 5% be Rs, x. Then, Money at 5% 800 Money at 8% — (1550-600) 50 | Reasoning & Aptitude MADE EASY 18. Ans. (c) (@ 2 9 Let the amounts invested be s, y,z respectively, 6 @s XX2x5_yxX3x5 100 100 ZxAx5 100 Then, x =10k, 20, ekaze ak & z= 5k So, x:y:2=10k Bk 5k = 90:20:15 =6:4:3. oooo Compound Interest 1. The difference in compound interest, and simple interest on a certain amount at 10% per annum at the end of the third year is Rs. 620. What is the principal amount? (a) Rs. 40000 (b) Rs. 12000 2, the compoundintereston acertansum at 162% for 3 years is Rs. 1270, the simple interest on the same sumal the same rale and forthe sarne period is: (@) Rs. 1200 fe) Rs. 1080 (b) Rs. 1165, (d) Rs. 1220 3, What is principal amount which earns Rs. 132 as ‘compound interest for the second year at 10% per annum? (a) Rs. 1000 (b) Rs. 1200 (c) Rs. 1320 (d) Rs, 1188 4, Asum of money amounts to As. 4624 in 2 years and to Rs, 4913 in 3 years at compound interest. The sum is (@) Rs. 4096 (0) Rs. 4335 (b) Rs, 4260 (d) Rs. 4360 5. Asum of money at compound interest amounts to thrice itselfin 3 years. In how many years willit be 9 times itsel!? 's-20000._______ {a 34» 6. Inhowmany years willa sum of Rs. 800 at 10% per annum compounded semiannually became Res, 926.107 i (b) 15 1 @) 25 2 1 4 ( 23 15 7. Tofind out the total compound interest on a sum of money after 5 years, which of the following informations given in the statements P and Q will be sufficient? P: The sum was Rs, 20000. Q:: The total amount of simple interest on the sum after 5 years was Rs. 4000. (@) Only P is sufficient (0) Only Qis sufficient (c) Either P or Qis suticient (d) Both P & Q are needed 8. The least number of complete years in which a sum of money put out at 20% compound interest willbe more than doubled is: 5 @ 6 1 9. Atroe increases annually by % th ofits height. By how much wil it increase after 2 years, if it stands today 64 cm high? (@) 72m (©) 75cm (b) 74cm (6) 8tem 10. The compound interest on a sum for 2 years is Rs. 832 and the simple interest on the same sum for the same period is Rs. 800. The difference between the compound interest and the simple interest for 3 years will be (@) As. 48 (0) Re. 66.56 (©) Rs.98.56 (0) None of these 11. A sum of Rs. 1100 was taken as a loan. This is to be repaid in two equal installments. |f the rate of interest be 20% compounded annually, then the | value of each installment is fa) Rs. 842 (b) Rs. 792 (©) Rs. 720 (a) Rs. 700 12, What annual payment will discharge a debt of Rs. 7620 due in 3 years at 162% per annum compound interest? (a) Rs. 2540 (bo) Rs. 3430 (c) Rs. 3260 (d) Rs. 3380 ooo Solutions 1. Ans. (d) Let the sum be Rs. x. Then, 4 cls o( wig) x 100, -(2t-4) Bix ~ (4000, 1000 xx10%3_3e 5) = Sx10x8_8r 100 10 (Cl)-« 3tx = 620 7000 or x= 20000. Hence, the principal amount is Rs. 20000. Alternative: RY RY os 700 2. Ans. (c) Let the sum be Rs. x. Then, [»(t+s2ta) f+ -( Ch 216 127% 2 ‘| ete © 216 270 or x= 1270x216 ey Thus, the sum is Rs. 2160. = 2160. Si-F{2160« = Rs. 1080. * Simple Interest & Compound Interest | 51 Ans. (b) Lt the principal at the end of first year be Rs. x Then, POT 6 132 or x = 1920. Now, let the original principal be Rs. P. Then, amount after 1 year Pxt0xt_11P =P+ 100 ~ 10 up MP at a= 1920 or SSALO s,120. ‘Ans. (a) Interest on Rs. 4624 for 1 year = Fis, (6083.50 ~ 5290) = Rs. 793.50 pate = (100278850) 1 a |= 62% 4 4624x1 ay =|" F599) lusing (o) Hence, n= 6 years, Ans. (b) Rate = 5% per half year. Let time = 2n half years = n years. 5 Then, son( 1 i) = 926.10 100, = (2 20, 3 2n=8 orn== years orn=5 years ale 926.10 _ 9261 (2) 800 8000 (20 52 | * Reasoning & Aptitude MADE Ensy 7. Ans. (a) 8 Clearly, both P and @ together are needed, Using P = f(a 70"'| and Q rate canbe calculated as Rs. 66.56, Total difference = Rs. (32 + 66.56) 1004000 Rate = (120% 4000) _ go, = Rs, 98.66 (Sam) 11. Ans. (c) - Let the value of each installment be Rs. x. 8. Ans. (b) Then, (PW. of Rs..x due 1 year hence) + (PW, of Rs. x due 2 years hence) = Rs. 1100. =1100 $0, x | 25x aor or 58. 28% _ 4499; 55x=96x 1100 6 36 9. Ans. (d) (22 rneaco® =(Jxt0)e= 102 . 12. Ans. (b) Hei oh afer 2years Let each installment be Rs. x, Then, 2 -ca(1e 255 em sy (cax2x2) (ars) (+ =(4 a —— {esx peglomas . 10. Ans. (c) yn Diff. in C.l. & SL. for 2 years = Rs, 32. (+535) Sl. for 1 year = Rs. 400, St on Rs. 00 for 1 year = R32. Gry soa eee aa ada 7620. ae = (2222) <0 20de + 252% + 216s = 7620 x 343 Ce px 1820%343 _ 599 Hence, diff. nC. and 8.1. for 3% year _ 762 = Sil on As. 832 + Amount of each installment = Rs.3430. CHAPTER Ratio Ratio is the relationship between the quantities of same kind. In ratio the quantities are compared as the multiple or parts of other quantities. If ratio is a : b then a is called antecedent and b is called consequent. ratio of the quantities is expressed after removing the common factor between the quantities. Generally, the ratio is useful in comparision. If the quantities A & B are compared & their ratio comes out asp:q We can say A/B = piq or A & B can be expressed as pK, qK respectively. This provides a scope of comparison in terms of multiples of p & q. In Equalities in the Ratio fa ratio is given as a: b & a quantity x is added in both antecedent & consiquent then atx a » pre Eitan a) » 2e% cB itasb ci) = pita=b ve will reverse (i) and (ii) was considering x = tx = -ve (i) and (i inequalit Ratio as a part of total value I two quantities A & B are in the ratio p : q then ALB. ; Bg PAR PKB = ak So A+B=(p+q)K go AL. Be i Aee ped o Az =P (AsB) = peg Pp SoA will be 5g part of total sum of the quantities similarly B will be er part of total sum of the quantities Example 1. Given that for two quantities a & b, 2a = 3b, What will be the ratio (4a + 5b) : (2a + 3b)? Solution: a) a= ab= 5 = 5 a= 3K,b=2K +5b _ 4x3K+5x2K ° Bat = BXGK+3xIK So the required ratiois 11 :6 Example 2. Wa:b=3:5&a:c=6:7 then whatis a:b:c? Solution, a:b=3:5 a:c=6:7 Inthis case the common quantity is ‘a’ so we will make a's value as const Sowe can say a: b= 6: 10 (multiplying by 2) anda:e¢=6:7 soa:b:c=6: 10:7 Example 3 10a? ab _ 10 = > what will be a: b Jab Bry What wll be a:b (@) 2:3 (0) 2:5 () 3:4 @3:7 Solution Wai+ab _ 10 Bab-bF 7 54 [© Reasoning & Aptitude or 10a?+ ab = 30ab- 10b? 5 Wat+ab _ 30ab-10b* ab ab = v0(Z}e1 ie w0-(2) b a a Assuming ae = 4r = 29-10() x or 10x? = 29x~ 10 = 10x’- 29x + 10=0 10, 25 = 35 10 2 5 = _ Hence option (b) Types of Ratio If two quantities A & B have their ratio as A:B=P:qthen * Duplicate ratio of A : Bis P*: g? «Triplicate ratio of A : Bis P®: g? * Sub-duplicate ratio of A: Bis JP: q, * Sub-triplicate of A: Bis YP: fq __Example 1. The ratio of two quantities A & B is 4; 9 what is the triplicate ratio of sub-duplicata ratio of A & B. Solution. The sub-duplicate ratio of A & Bis V4: J = 2:3 The triplicate ratio of 2 : 3 is 2°: 33 ae te Proportion Proportion is comparison of two equal ratio when the two ratios are equal then all the quantities Comprising the ratios are called in proportion ie then a, b, c & dare in proportion & ‘dis a bod called as fourth proportion of a, b & ¢ ifthree quantities a, b, c are taken then a ’ b, b is called as second proportion or mean proportion of a & c. b ‘phere c is called as third proportion of a & MADE ERsy so bi = ac o b= Jac 80 mean proportion is the geometric mean of the ‘wo quantities. Here the quantities b and ¢ will be in‘continued proportion’ similarly fa, b, ¢ dare in continued proportion then or in other way we can say that if the terms are in GP. then they will be in continued proportion. Example 1 Find out the mean proportion of 3 & 27 also find out their third proportion. Solution. For mean propottion 3, x, 27 are in proportion 3x me x 27 = = 3x27 orr=9 So mean proportion of 3 & 27 is 3 For third proportion 3, 27, x are in proportion alee, re oy = ar = 272 = x = 243 So third proportion of 3 & 27 is 243, Applications of Proportion Proportion is very much useful in removing the Complexity of the problems in ratios. There are some basic rules that has to be taken care: iff, b, ¢, dare in proportion a ie. . d 24 scomponendo ie nen 2 dividend rue) c+d c-d (componendo-diviceendo rule) MADE EASY * Ratio, Proportion & Variation | 55 Variation or AB const. 80 AB, = A,B, = A.B, =..A\b, = K The basic meaning of variation is respective change. Inarithmetic, use of variation makes problems very easy toapproach, Itis the relationship bewteen the changes of two quantities. Let us assume there are two quantities A & B if any change is done’A’ & that change results the change in B then the quantities are called in variation. One thing has tobe remembered the change we are counting is muttile change not the differential change i.e. If a quantity is 60 ‘and then it becomes 80 then the change will not be taken 5 (+20). It willbe taken as 80/60 = 4/3, * Direct variation : For two quantities a & b if the nature of changes are similar then they will be directly varying ie it aed then if 'a' becomes twice then ‘b’ will also be twice, Ifa becomes 2/3 times then b will also be 2/8 times. Example 1 Two quantities A & B ate varying directly. Initially, Awas 30 & 8 was 50 what has to be the value of B it Abecomes 24? Solution. Here AaB Change in A is 24/30 times = 4/5 times So the change in B will be the same So B will be 4/5 times of 50 = 4/5 x 5 In other way we can make it as. AeB o A= BK or AJB = constant = > 24 So x50 ve 5x = r= 4 Inverse Variation When in variation the nature of changes in the two quantities are inverse then the quantities will be in inverse variation? ie Ae then A ax ols Example 1. ‘Acar running with some speed increases ils speed, to 4/3 times & reaches its destination 30 minutes earlier. What time it will take iit runs with original speed? Solution. Here we know that speed « t/time So we can use variation property Let us assume the initial speed as 'S' & initial time ast Now by the property of inverse variation (speed), x (ime), = (speed), x (time), Sxt= fsx (time), 3 = (time), = St given that (time), - (time), = 30 mins. 3 t t 2% = $= 80 ort = 120mins So the car will take 120 mins with original speed Compound Ratio & Concept of Partnership I there are two ratio @ : b and ¢:dthen their compound, ratio will be a x ¢: b xd. This leads to the concept of partnerships the main part of the partnership is the division of profit between the partners, The profit among the partners is always Givided in the compound ratio of investment ratio and, time of investment ratio i. Ifthe partners’ investment ratio is P, :P, :P, & the ratio, of their investment time is t, : t;:1, then the ratio of profit = Pt, P,t,: Pt, So there may be three cases of the profit sharing, Case 1 If the ratio of investment is unityi.e. the invest equal amount then Py:P,:Pyzdiits4 So ratio of profit sharing will be t, : Example 1 Itthree persons A, B, C invests rupees 20000 in a business for 3 years, 2 years & 5 years respectively then what will be the ratio of their profits. 56 | © Reasoning & Aptitude MADE EASY Solution. @ 2 (©) 20 Since the investment is same for all the persons so (0) 40 @) 50 the profit will be divided in the ratio of Ans. (c) 1xB:1x2:1%5 a ie, 3:2:5 will be the profit ratio. Case 2. IF the investment time is same for all the persons ie.t:ty:t)= 12121, then the profit will be shared in the ratio of their investmentie, the ratio of profit will be PLP, Example 1. A, B, C invest rupees 10000, 18000 & 20000 respectively in a business for 2 years each what will be the ratio of their profit? Solution. Since the investment time is constant. So profit will ‘be shared in the ratio of their investment i.e. ratio of profit 10K :15K:20K 2:3:4 4 Case 3. When the investments variable we have to look for the effective investment vali for the profit sharing, Example 1. ‘A,B, C invested rupees 10000, 15000 & 20000 respectively. Next year A & B invested 5000 more while C withdrew 10000 rupees. What will be the ratio of their profit after 3 years? Solution. Here we will see the effective investment value of A, B&C respectively. Effective Investment Value of A = 10K+15K+15K = 40K 15K + 20K + 20K = 55K EVVofB EIofC = 20K +10K+10K 40k So ratio of profit will be 40K:55K:40K =8:11:8 Example 2. ‘A mixture contains alcohol and water in the ratio: 4:1 by adding 10 litre of water the ratio becomes 2:1. Find out the amount of alcohol in the mixture The amount of alcohol = 4 K ‘The amount of water = K. aK 2 By adding water 55 = 5 = 4K =2K+20 => 2K =0ork=10 So amount of alcohol = 4 x 10 = 40 litre Example 3. Tho present ratio of ages of Ram & Shyam is 5: 4. 18 years ago the ratio of their ages was 16: 11. The present sum of their ages is (@) 90 years (©) 105 years (©) 80 years (@) 110 years Ans. (a) Let the ages be 5 K, 4K 5K-18 46 4K218 = 77 2K = 10 So sum of the ages = 60 + 40 = 90 Hence option (a) Example 4. The income of Ram and Shyam are in the ratio 2:3 and their expenditures are in the ratio 3: 5. reach saves AS. TOOO then Ham's income ls (a) 8000 (b) 6000 (c) 5000 (a) 4000 Ans. (d) Let income be 2x, 3 and expendilure be 8y, 8y So x= 2000 So income is 4000 Hence option (a) goa Cy) Solved Examples 1. Divide Rs. 500 among A, B, C and D so that A and B. together get thrice as much as C and D together, B gets four times of what C gets and C gets 1.5 times. 2s much as D. Now the value of what B gets is (@) 900 (b).75 (ce) 125 (d) None of these | | MADE EASY * Ratio, Proportion & Variation | 57 Ans. (a) (a) Rs, 3000 (©) Rs. 4000 A+B+C+D=500 (©) Rs. 6000 (@) Rs. 9000 Here A+B =3(C +D) Ans.(c) $0 4(C +D) = 500 Let incomes are Sx : 2xcr C+D=125 expenditures are Sy : 3y A+B=375 then also B = 4C and C = 1.5D r~5y = 1000 (1) also c+o 2x~ By = 1000 (2) 25D from (1) and (2)we get D=50,C=75 6x 10y = 2000 2B =300 3000 2. If 4 examiners can examine a certain number of co y= 1000, x= 2000 answer books in 8 days by working 5 hours a day, for how many hours a day would 2 examiners have to work in order to examine twice the number of ‘answer books in 20 days, @ 6 () 7 (8 (9 Ans. (c) Man. Day. Hour = constant =4.8.5=160 Next > Man. day, hour = 160 2 (twice as earlier) 2,20.h= 160x 2 h=8 3. Inamixture of 40 litres, the ratio of milk and wateris 4:1. How much water must be added to this mixture 80 that the ratio of milk and water becomes:2: 3? (@) 20litres: (b) 22 itres (©) 40 litres (A) 30 litres Ans. (c) Let wateris x 4xrx= milk : water 32:8 = dv: x (since total 40 Iters) 32 2 ary ge Alive Now 4. The ratio between two numbers is 3 : 4 and their LCMis 180. The first number is: @ 0 (b) 45 (©) 5 da Ans. (b) Let number are 3x and 4x 8x x dy = HOF x 180 Clearly HCF will be x. 12x? = x x 180,x = 15 numbers will be 45 : 60 5. The incomes of A and are in the ratio 3 : 2 and their expenditures are in the ratio5:3. Ifeach saves Ais. 1000, then, A’s income is As income is 6000 Divide Rs. 680 among A, B and C such that A gets 2/8 of what B gets and B gets 1/4th of what C gets. Now the share of C is? (a) Ps. 480 (b) Rs. 300 (c) Rs. 420 {@) None af these Ans. (a) 680=A+B4+C 2 1 A= 3B andB= 7C, 4B=C ‘The students in three batches at Made Easy are in the ratio 2:3: 5. 20 students are increased in each batch, the ratio change to 4 :5: 7. Thetotal number of studlenisin the thee batches before the increase were, (@) 10 (b) 90 (©) 100 (d) 150 Ans. (c) Let students are 2r : Sx : 5x now ‘According to the given condition Br + 20:8v-420: Gx + 20=4:5:7 x=10 20: 30:50 = 2:3: 5 and after adding 20 40: 50:70 The speeds of three cars are in the ratio 2:3: 4. The ratio between the times taken by these Cars to travel the same distance is (@) 2:3:4 (b) 4:3:2 (©) 4:3:6 (0) 64:3 58 | © Reasoning & Aptitude MADE EASY Ans. (d) (2) 48 (0) 72 Speeds are in the ratio 2: 3:4 © © (@) None of these feed Ans. (d) ratio of time taken will be 3° Colne are 122 : 10:7 Sole a i 1,7 12 tis given that 12+ 10rx 5+ 3x = 75 9. After an increment of 7 in both the numerator and denominato. a ecten cargo 10 Fraime 22002 72 original fraction. @) sre (b) 7 (ee : (c) 25 (@) 38 13, A varies jointly with B and C directly and Ans. (c) A= 6when B = 3, C=2;find Awhen B = 5, C=7. x+7_3 (ai75 (0) 35 ye7 4 (©) 70 (d) 105, 4x +28 = By 421, de+7 = 3y, Ans. (b) j only x: y = 2/5 satisfies the given condition, Let A =k (BC) then | 10. | FRs, 58s divided amiong 150 children such that each S0.k iv a = | girl and each boy gets 25 p and 50 p respectively Aak(BC) | Then how many girs are there? 1x 6x7 (a) 52 (b) 54 (©) 8 (d) 6 14, If x varies as y directly, and as z inversely and Ans. (c) x= 14, when y = 10;find z when x = 49, y = 45, Let the number of gis and boys are x and y then @ 14/10 0.25x + 05y = 58 (1) () 10 also (© 104 reyes 180 ~ ol) (a) Cannot be determined from (1) and (2) we get Ans. (d) OSr+y= 116 ko x+y = 150 ot eT So, 0.5x = 34, x= 68, y = 82 ‘According to given values 11, A mixture contains milk anc water in the ratio 5:1 eee On adding 5 litres of water, the ratio of milk to water becomes § : 2, The quantity of milk in the mixture is: (@) 16 lites (©) 25 litres (©) 32.5iitres (6) 22.75 lives Ans. (b) But value of k, is not given, Let milk andl water are Sx : x Hence Z cannot be determined. owe) 15. A cask contains a mixture of 49 litres of wine and 452 water in the proportion 5 : 2. How much water must fox = Sx 425 be added to it so thatthe ratio of wine to water may Sx = 25, x=6 be7:4? milk : water = 25: 10 fa) 35 ) 6 7 (d) None of these 12, Vijay has coins of the denomination of Re. 1, 60 p Ans. (b) and 25 pin the ratio of 12: 10: 7. The total worth of Let wine and water are 6x and 2x litres the coins he has in Rs. 75. Find the number of 25 p onan coins that Vijay has Sr =35, Qe=14 * Ratio, Proportion & Variation | 59 MADE EASY 357 Tatw 4 98 + 7w= 140 Tw = 42,we gooo Ratio & Proportion (i) Practice Exercise: ! 1. Find a fourth proportional to the numbers 60, 48, 30, (a) % (b) 24 (c) 48 (d) None of these 2. Find a third proportional to the numbers 4, 42. (@) 441 (b) 541 (© 641 (d) None of these 3. The prices of a scooter and a television set are in the ratio of 3 : 2. If a scooter costs Rs. 600 more than the television set, then the price of television set is: (@) Rs. 1800 (©) Rs. 1200 (©) Rs. 2400 (@) None of these 4, WA: B=7:8andB:C=9: 11, then A:B :Cis equalto (2) 55:45:63 (©) 63:45:55 (©) 456355 (2) None of these 5. The ratio of money with Anju and Sanju is 4:5 and, that with Sanju and Manju is 5: 6. if Anju has Rs. 280, then the amount of money Manju has (a) Rs, 320 (b) Rs. 420 (©) Rs. 640 (6) None of these 6. The ratioof present ages of Suresh and Mahesh is 7:5. Ifatter 6 years their ages wil be in the ratio of 4:3, the present age of Mahesh is (a) S2years (©) 36 years (©) 30years (d) None of these 7. Two numbers are in the ratio of 6 :7. If 2 be subtracted from each, they are in the ratio of 35 : 59. Find the difference of the two numbers. (@) 48 (6) 82 (c) 24 (d) None of these 8. The number that must be added to each of the numbers 8 21, 13 and 31 to make the ratio of first ‘two numbers equal to the ratio of last two number is fa) § (b)7 9 (d) None of these 9. Amixture contains alcohol and water in the ratio of 12: 5. On adding 14 litres of water, the ratio of alcohol to water becomes 4: 3. The quantity of alcohol in the mixture is (@) 18 litres (b) 42 tres, (©) 26 litres (d) None of these oooo Solutions 4. Ans. (b) Lets be the fourth proportion, then 60 _ 38 60:48:38 .x ons = 0x3 xe = 24, 0 2. Ans. (a) Let x be the third proportion, then The price of a television set by _ 2600 “a-b 3-2 4. Ans. (b) Here, n, = 7, ny=9,d, = Sand det A:B:C=(n,xn,}: (d,xn,) : (d,xd,) =(7%9):(5x9):(5« 11) = 63: 45: 56. 5. Ans. (b) We have, A:B=4:5andB:C = 5:6. Here, n, = 4, n,= 5, d,= Sand d= 6. A: B:C=(n,xn,):(dpen,) : (Aya) =(4%5):(5x5):(5%6) = 20:25: 3001, 4:5:6 = Rs, 1200. 60 | © Reasoning & Aptitude Thus, ratio of money with Anju, Sanju and Manju is 4:5: 6. Since Anjuhas Rs. 280, the amount of money Manju has 6. Ans. (c) We have, a: b=7:5,0:d=4:3 and x= tx(c-d) Th nt age of Mahesh = — fe present age of Mahesh = _5x6x(4-3)_ ~ (e-Bay = 90 Years 7. Ans. (c) We have, a:b =5:7,¢:d=95: 59 and x= 25 ax(d-c) ‘ad—be 5x25x(59-35) (5x59-7x%) The first number = 60. bx(d-o) ad—be and, the second number = The difference of two numbers = 84-60 = 24 8. Ans. (a) Here, a=8,b=21,c= 13andd=31 The required numbor be~ad +d)-(b+e) 21x13-8x31 (8+31)-(21+13) 9. Ans. (b) Ratio of alcohol and water 12: 5 Let their qualities be 12x and 8x respectively. Alter adding the litres of water ratio becomes 4:3 (ore Beata ~ RIN ele 7 Quantity of alcohol = 125 42 litres, goog |. A, B and C start a business. I! the ratio of their mADE EASY Partnership Practice Exercise: | ‘Amit, Nitin and Ravindra entered into a partnership. Amitinvested Rs. 16000 for9 months. Nitin invested Rs, 12000 for 6 months and Ravindra invested Rs, 8000 for 12 months. At the end of a year there was a profitof Rs, 26000. Find the share of Nitin in the profit (@) Rs. 8000 (b) Rs. 7500 (©) Rs. 6000 (@) None of these | | A starts business with Rs, 3500 and § months after B joins A as his partner. Aller a year the profits are divided in the ratio of 2: 3. How much did B contribute? (@) Rs. 7000 (b) Rs. 11000 (©) Rs. 9000 (d) None of these A, B and C invested capitals in the ratio 3: 5:9; the timing of their investments being in the ratio 2: 3: 1. In what ratio would their profits be distributed? fa) 2:5:3 (0) 7:5:3 (b) 3:2:5 (d) None of these periods of investments are 2 : 3: 6 and their profits are in the ratio of 4 : 5 : 6, then the ratio of capitals of A, Band Cis (a) 6:8:10 (©) 10: 12:6 fo) 12:10:6 (d) None of these and B slarled a business with initial investments inthe ratio §: 7. If after one year their profits were in the ratio 1 : 2 and the period for A's investment was 7 months, B invested the money for (a) months ) 2 (c) 10months (d) 4 months A, B, Center into @ partnership with shares in the tatioZ:2:8. ater 4 months, Aincroase his share by 50%. Ifthe total profit at the end of one yearbe | es, 21600, then B's share in the profit is (@) Rs.2100 (©) Rs. 2400 (©) Rs.3600 (@) Rs. 4000

You might also like